Torts 2 Case Briefs

87
TORTS 2 CHAPTER 8 - DAMAGES...................................................... 3 INTRODUCTION...................................................................3 PERSONAL INJURY................................................................4 RODRIGUEZ v. MCDONNELL DOUGLAS CORP..........................................4 MCDOUGALD v. GARBER..........................................................5 HAYNES v. YALE—NEW HAVEN HOSPITAL............................................7 WRONGFUL DEATH AND SURVIVAL ACTIONS............................................8 ASPINALL v. MCDONNELL DOUGLAS CORP..........................................10 McDAVID v. UNITED STATES....................................................10 OTHER COMPENSATORY DAMAGE ISSUES..............................................12 PUNITIVE DAMAGES..............................................................13 STATE FARM MUTUAL AUTO. INS. CO. v. CAMPBELL................................13 CHAPTER 9- AFFIRMATIVE DEFENSES BASED ON PLAINTIFF’S CONDUCT............15 INTRODUCTION..................................................................15 CONTRIBUTORY AND COMPARATIVE NEGLIGENCE.......................................16 HOFFMAN v. JONES............................................................16 WASSELL v. ADAMS............................................................17 FAILURE TO AVOID CONSEQUENCES; FAILURE TO MITIGATE DAMAGES....................18 DARE v. SOBULE..............................................................18 HUTCHINS v. SCHWARTZ........................................................19 ASSUMPTION OF THE RISK........................................................21 BENNETT v. HIDDEN VALLEY GOLF AND SKI, INC..................................21 IMPUTED CONTRIBUTORY FAULT....................................................22 CONTINENTAL AUTO LEASE CORP. v. CAMPBELL....................................22 WHITE v. LUNDER.............................................................24 CHAPTER 10- OTHER AFFIRMATIVE DEFENSES..................................25 STATUTES OF LIMITATIONS.......................................................25 JOLLY V. ELY LILLY & CO.....................................................25 FELTMEIER v. FELTMEIER......................................................26 STATUTES OF REPOSE............................................................28 BRADWAY v. AMERICAN NATIONAL RED CROSS......................................28 EXPRESS ASSUMPTION OF RISK....................................................29 HOLZER v. DAKOTA SPEEDWAY, INC..............................................29 CHAPTER 11 – COMPARATIVE RESPONSIBILITY IN MULTIPARTY LITIGATION........31 CONTRIBUTION AND INDEMNITY....................................................31 BROCHNER v. WESTERN INSURANCE COMPANY.......................................31 JOINT AND SEVERAL LIABILITY...................................................32 1

Transcript of Torts 2 Case Briefs

Page 1: Torts 2 Case Briefs

TORTS 2

CHAPTER 8 - DAMAGES................................................................................................................................................ 3INTRODUCTION............................................................................................................................................................................. 3PERSONAL INJURY........................................................................................................................................................................ 4

RODRIGUEZ v. MCDONNELL DOUGLAS CORP.....................................................................................................................................4MCDOUGALD v. GARBER............................................................................................................................................................................... 5HAYNES v. YALE—NEW HAVEN HOSPITAL.........................................................................................................................................7

WRONGFUL DEATH AND SURVIVAL ACTIONS.................................................................................................................... 8ASPINALL v. MCDONNELL DOUGLAS CORP.......................................................................................................................................10McDAVID v. UNITED STATES....................................................................................................................................................................10

OTHER COMPENSATORY DAMAGE ISSUES........................................................................................................................ 12PUNITIVE DAMAGES................................................................................................................................................................. 13

STATE FARM MUTUAL AUTO. INS. CO. v. CAMPBELL...................................................................................................................13

CHAPTER 9- AFFIRMATIVE DEFENSES BASED ON PLAINTIFF’S CONDUCT...............................................15INTRODUCTION.......................................................................................................................................................................... 15CONTRIBUTORY AND COMPARATIVE NEGLIGENCE.......................................................................................................16

HOFFMAN v. JONES....................................................................................................................................................................................... 16WASSELL v. ADAMS...................................................................................................................................................................................... 17

FAILURE TO AVOID CONSEQUENCES; FAILURE TO MITIGATE DAMAGES...............................................................18DARE v. SOBULE............................................................................................................................................................................................. 18HUTCHINS v. SCHWARTZ........................................................................................................................................................................... 19

ASSUMPTION OF THE RISK..................................................................................................................................................... 21BENNETT v. HIDDEN VALLEY GOLF AND SKI, INC.........................................................................................................................21

IMPUTED CONTRIBUTORY FAULT....................................................................................................................................... 22CONTINENTAL AUTO LEASE CORP. v. CAMPBELL.........................................................................................................................22WHITE v. LUNDER......................................................................................................................................................................................... 24

CHAPTER 10- OTHER AFFIRMATIVE DEFENSES................................................................................................ 25STATUTES OF LIMITATIONS.................................................................................................................................................. 25

JOLLY V. ELY LILLY & CO............................................................................................................................................................................ 25FELTMEIER v. FELTMEIER........................................................................................................................................................................ 26

STATUTES OF REPOSE.............................................................................................................................................................. 28BRADWAY v. AMERICAN NATIONAL RED CROSS...........................................................................................................................28

EXPRESS ASSUMPTION OF RISK............................................................................................................................................ 29HOLZER v. DAKOTA SPEEDWAY, INC...................................................................................................................................................29

CHAPTER 11 – COMPARATIVE RESPONSIBILITY IN MULTIPARTY LITIGATION.....................................31CONTRIBUTION AND INDEMNITY........................................................................................................................................ 31

BROCHNER v. WESTERN INSURANCE COMPANY...........................................................................................................................31JOINT AND SEVERAL LIABILITY............................................................................................................................................ 32

KAEO v. DAVIS................................................................................................................................................................................................. 32BROWN v. KEILL............................................................................................................................................................................................. 33VARELA v. AMERICAN PETROFINA COMPANY OF TEXAS, INC................................................................................................34

Partial Settlements.................................................................................................................................................................... 35MCDERMOTT, INC. v. AMCLYDE..............................................................................................................................................................35

APPLICATION OF COMPARATIVE RESPONSIBILITY TO CAUSES OF ACTION OTHER THAN NEGLIGENCE.. .37

1

Page 2: Torts 2 Case Briefs

BONPUA v. FAGAN......................................................................................................................................................................................... 37

CHAPTER 12 - IMMUNITIES...................................................................................................................................... 39A. GOVERNMENT IMMUNITY................................................................................................................................................ 39

1. STATE AND LOCAL GOVERNMENT.................................................................................................................................................39HICKS v. STATE................................................................................................................................................................................................................ 39

2. FEDERAL GOVERNMENT.....................................................................................................................................................................42a. FEDERAL TORT CLAIMS ACT.............................................................................................................................................................................. 42

UNITED STATES v. GAUBERT...............................................................................................................................................................................42b. JUDICIALLY CREATED IMMUNITY................................................................................................................................................................... 43

UNITED STATES v. JOHNSON............................................................................................................................................................................... 433. IMMUNITY OF GOVERNMENTAL OFFICERS AND EMPLOYEES.........................................................................................444. THE “PUBLIC DUTY” DOCTRINE...................................................................................................................................................... 44

RISS v. CITY OF NEW YORK........................................................................................................................................................................................ 44B. FAMILY IMMUNITIES.......................................................................................................................................................... 45

PRICE v. PRICE................................................................................................................................................................................................. 45BROADBENT v. BROADBENT................................................................................................................................................................... 46

CHAPTER 13 - MEDICAL MALPRACTICE............................................................................................................... 47PROFESSIONAL STANDARD OF CARE.................................................................................................................................. 47

MELVILLE v. SOUTHWARD........................................................................................................................................................................47HARNISH v. CHILDREN’S HOSPITAL MEDICAL CENTER.............................................................................................................49

CHAPTER 14 – COMMON LAW STRICT LIABILITY............................................................................................. 51ABNORMALLY DANGEROUS ACTIVITIES............................................................................................................................ 51

RYLANDS v. FLETCHER............................................................................................................................................................................... 51SEIGLER v. KUHLMAN.................................................................................................................................................................................. 53

CHAPTER 15 - PRODUCTS LIABILITY.................................................................................................................... 54EMERGENCE OF STRICT TORT LIABILITY.......................................................................................................................... 54

GREENMAN v. YUBA POWER PRODUCTS, INC.................................................................................................................................54DEFECT.......................................................................................................................................................................................... 55

GRAY v. MANITOWOC CO., INC................................................................................................................................................................ 55PHILLIPS v. KIMWOOD MACHINE CO...................................................................................................................................................56FELDMAN v. LEDERLE LABORATORIES..............................................................................................................................................57POTTER v. CHICAGO PNEUMATIC TOOL CO......................................................................................................................................59

2

Page 3: Torts 2 Case Briefs

CHAPTER 8 - DAMAGES

INTRODUCTIONA. Categories of Damages: - There are 3 main categories of damages: nominal damages, compensatory damages, and punitive damages - The cause of actions for some intention torts is complete without proof of actual damages. When a plaintiff wins

such a case without proving any damages, the court awards nominal damages, which may be defined “as a trivial sum of money awarded to a litigant who has established a cause of action but has not established that he is entitled to compensatory damages.”- Restatement 2d. Designed to vindicate a right.

- Compensatory Damages- the damages awarded to a person as compensation, indemnity, or restitution for harm sustained by him.”- Restatement 2d. The basic theory behind compensatory damages is restoration of the plaintiff to his or her pre-injury condition, to the extent that an award of money can do that. In causes of action based on strict liability and negligence, the plaintiff must prove compensatory damages as a part of the prima facie case. Most of the litigation occurs under compensatory damages.

- Punitive Damages- “the damages, other than compensatory or nominal damages, awarded against a person to punish him for his outrageous conduct and to deter him and other like him from similar conduct in the future”- Restatement 2d. – To punish and deter the wrongdoer from heinous conduct (some type of outrageous damages)

B. Compensatory Damages For Physical Harms to Property - For an invasion of land, the basic measure of recovery is any diminution in the land’s value caused by the

invasion, or at the plaintiff’s election, the reasonable costs of restoration, plus the value of the plaintiff’s loss of use of the land (the rental value.)

- When a chattel has been converted or destroyed, the basic measure of damages is its entire value at the time and place of the tort (fair market value).

- When a chattel has been damaged, but not destroyed, the basic measure of damage is the diminution in value caused by the tort (repair costs are often relevant, but are not the final measure).

- In all of these cases, the court may also award “consequential” or “incidental” damages. o Business profits (if proved with sufficient certainty)o Example of trespass to land including damages for injury to person or chattels caused by the trespasser.

C. Compensatory Damages In Personal Injury Cases- Three main types of personal injury compensatory damages:

1. Medical (and related) expenses (past and future)- pecuniary damages/ special damages because they are capable of economic evaluation, even though future losses are uncertain. Past medical expenses are established by the hospital, doctor bills. Future losses generally require expert testimony to establish what kinds of expenses are likely to be incurred and how much the expenses are likely to be.

2. Loss of earning capacity (past and future)- includes loss of past earnings pecuniary damages/special damages . Typically past loss of earnings are established by projecting what the plaintiff was earning at the time of the injury through the date of the trial. However, this may require expert testimony if there would have been a change in circumstances, such as promotion, industry layoffs, etc. Future earning capacity loss is typically proven through the use of experts. These may include a variety of types of experts including economists, labor specialists, rehabilitation experts, etc.

3. Physical and mental pain and suffering (past and future)- non-pecuniary damages, referring to pain and suffering that can’t be easily measured in dollars. – This is where juries have the most leeway because there really isn’t anything to follow. These are the type of damages that are sometimes known as general damages, are those not susceptible of economic evaluation. A variety of evidence is used to give the jury a basis for making a finding on the issue of general damages.

a. Wide variety of evidence-i. Plaintiff testimony

ii. Family members

3

Page 4: Torts 2 Case Briefs

iii. Doctors, therapists, counselors, are often part of P’s caseiv. Photographs v. Videos

vi. Charts and diagramsvii. Computer graphics

viii. Thermographic imaging- supposed to show areas of pain according to heat/inflammation

PERSONAL INJURY

RODRIGUEZ v. MCDONNELL DOUGLAS CORPCalifornia Court of Appeals, 1978

87 Cal.App.3d 626, 151 CalRptr. 399Page 328 in casebook

Procedural History- At the conclusion of the trial in 1975, McDonnell Douglas was exonerated but found both of the contractors liable for damages to Rodriguez and his wife in the value of over 4.5 million dollars.

Facts- Plaintiff, Richard Rodriguez, was a sprinkler fitter’s apprentice for a fire protection company that subcontracted to McDonnell Douglas for a hangar modification project. Due to the negligence of two other contractors, a 630 pound pipe fell on Rodriguez, causing him to become a triplegic. He would never be able to walk again. He suffered massive brain damage, a fractured spine and irreparable damage to his spinal cord. He was subsequently hospitalized many times and had many surgeries as well. Also lost kidney, bowl, and bladder functions.

Issue(s)- Did the trial court err in overruling the defendants’ motion for a new trial on the ground that the size of the award suggested that the jury acted on passion and prejudice?

Holding- No. The awards to the plaintiff were reasonable given the grievous nature of the injuries Rodriguez received. The defendants produced no evidence that the awards the jury gave reflected passion and prejudice. The award did not “shock the conscience.”

Rule(s)- The determination of damages is primarily a factual matter on which the inevitable wide differences of opinion do not call for the intervention of appellate courts. An appellate court, in reviewing the amount of damages, must determine every conflict in the evidence in respondent’s favor and give him the benefit of every reasonable inference. An appellate court may not interfere with an award unless ‘the verdict is so large that, at first blush, it shocks the conscience and suggests passion, prejudice, or corruption on the part of the jury.

Analysis-

Disposition- Affirmed

Notes from Class- - What rule did the court apply in reviewing the damage award to the plaintiff?

o Appellate may not interfere with the size of the jury’s award unless it is so large that it shocks the conscience and suggests passion, prejudice or corruption

- Relatively rare that a judge will overturn an award- The jury is not told what other jury’s award other people… so there is no “going rate” when it comes to giving

awards to plaintiffs (especially when it comes to pain and suffering, there is no real rule)

4

Page 5: Torts 2 Case Briefs

- An award could shock the conscience if it is too high or if it is too low. There is a lot of strategizing for where the case is filed depending on the community you are in (juries are different)

- How does one establish loss of earning capacity?o Generally an expert witness that is an economist o The younger the person is who has a long work history, then the award is going to be less

- Earning Capacityo What is the nature of the loss?

What the person could have earned - How does the D deal with testimony of the P’s economist?

o Sometimes the best defense is not to say anything, to keep him from talking, unless you can undermine their data, or inconsistencies in their facts

- What is the obligation of spouse with respect to “attendant care”? o There is not one as a wife to render 24 hours in a day assistance. This would be unreasonable. She

shouldn’t have to require care around the clock. - Inflation

o This is taken into account because over time inflation changes and things are not worth the same value as they were before.

- Discount Rateo Compounding or calculation of a future valueo 336-337 o Economist picks out the appropriate discount rate based on his experience and education

- Present value- Consortium

o Majority view- In a non death claim, the spouse of the injured party may have a claim for loss of consortium (loss of sexual relationship, companionship, love and other services

o Generally applies to spouseso Minority view-Some states allow children to recover for loss of society, companionship, or guidance in

non-fatal cases and some allow parents to recover loss of consortium for injuries to children - Reducing awards

o New trial (complete or partial) Remittur – the award was excessive, reduce the award, can terminate the case and accept the

reduced award or it can be remanded for a new trial Additur- the award was too little, the jury didn’t award enough, allows the option to bring the

award up to a certain amount of money. The defendant has the option to go back to trial or not. - Present value- pecuniary damages are generally reduced to present value. However, non-pecuniary damages are

not.

MCDOUGALD v. GARBERNew York Court of Appeals, 1989

73 N.Y.2d 246, 538 N.Y.S.2d 937, 536 Page 337 in casebook

Procedural History- Action was brought by McDougald and her husband against the defendants for malpractice. Jury awarded Emma McDougald $9,650,102 in damages for conscious pain and suffering and for the loss of the pleasures and pursuits of life. Her husband was awarded $1.5 million for the loss of his wife’s services. The trial judge reduced her reward to $4,796,728 by striking the entire award for future nursing care and reducing the award for conscious pain and suffering and for the loss of the pleasures and pursuits of life. On cross appeal, the appellate division affirmed and later granted defendants leave to appeal to the N.Y. Court of Appeals.

5

Page 6: Torts 2 Case Briefs

Facts- Plaintiff, Emma McDougald, underwent a C-section and tubal ligation at N.Y. Infirmary. Defendant, Garber, performed the surgery and defendants, Armengol and Kulkarni, provided the anesthesia. During the surgery, McDougald suffered oxygen deprivation which resulted in severe brain damage and left her in a permanent comatose condition.

Issue(s)- Did the trial court err in instructing the jury that P’s awareness was irrelevant to their consideration of damages for loss of enjoyment of life and in directing the jury to consider that aspect of damages separately from pain and suffering? Does the P have to have some level of awareness? Should loss of enjoyment of life be considered a category of damages separate from pain and suffering?Holding- Yes the trial court erred in instructing the jury that P’s awareness…There must be some cognitive awareness in order to recover nonpecuniary damages for the loss of enjoyment of life. However, the fact finder does not have to determine degrees of cognitive awareness in order for a plaintiff to recover. In addition, just because enjoyment of life and pain and suffering can be distinguished, the two do not need to be made into separate categories.

Rule(s)-1. Cognitive awareness is a prerequisite to recovery for loss of enjoyment of life. There must be “some level of

awareness” in order for a plaintiff to recover. 2. Punitive damages, those which have no compensatory purpose, are prohibited unless the harmful conduct is

intentional, malicious, outrageous, or otherwise aggravated beyond mere negligence.

Analysis- Jury should not make separate awards for pain and suffering and loss of enjoyment of life. Some states do hold that a P need not be conscious to recover damages for loss of enjoyment of life (Hedonic Damages). Some states also permit a separate award for loss of enjoyment in addition to the award for pain and suffering.

Disposition- Remanded for a new trial on the issue of nonpecuniary damages for Emma McDougald

Notes from Class- Compensatory purpose v. punitive purpose- compensation v. punishment/deterrence - Does the injured victim (P) have to be cognitively aware of pain and suffering in order to recover damages for

such pain and suffering?o Must be some conscious awareness o Not an exact level of consciousness, however, there must be some awareness of pain and suffering

- Does the plaintiff have to be aware of the loss of pleasures and pursuits of life (hedonic damages)?o This Court said yes… you have to be aware that you have lost something o No standard of awareness though

- Should the loss of enjoyment of life be treated as a separate item of damages from pain and suffering?o No, these are really one item. It would distort the amount of the award and possibly bump it up.

Generally, courts do not treat them as separate line items. o Not a damage that is measurable in economic terms. It is all about what the loss means to the plaintiff. If

they are not aware of the loss, then it does not mean anything to them. - Disagreement among states:

o Some allow recovery for loss of enjoyment (hedonic) damages whether or not the P is aware of the loss- A second issue is whether pain and suffering should be treated separately from hedonic damages in making an

award o States are split on this

- Not all legal jurisdictions make general awards for pain and suffering. Other places like Europe do not give out these awards like America does.

- Notes 4 and 5

6

Page 7: Torts 2 Case Briefs

HAYNES v. YALE—NEW HAVEN HOSPITALSupreme Court of Connecticut, 1997

243 Conn. 17, 699 A.2d 964Page 345 in Casebook

Procedural History- Decedent’s daughter, Susan Haynes, brought a wrongful death suit against Perrier, the other driver, and received a settlement of $20K from his insurance agency. She then received $630K from her own insurance agency. She then filed a malpractice claim against the hospital and the surgeon. The trial court granted the defendants’ motion for summary judgment on the ground that the plaintiff had already received full compensation for the harm suffered by her decedent.

Facts- Barbara Freeman, plaintiff’s decedent, was injured when she was in a car accident with Alan Perrier. She was taken to Yale-New Haven’s emergency room and was treated for a fractured left leg and pelvis. After an hour and a half, the emergency room doctors noticed she was experiencing “an expanding abdominal girth.” She was taken into surgery, and the doctor noticed large amounts of blood in her abdomen due to a laceration of the spleen. During the surgery she went into cardiac arrest and died. She was awarded $20,000, which was the limit of his insurance coverage. Freeman had purchased an additional $900,000 worth of coverage for herself. An arbitration panel fixed the total damages at $650,000 and the insurance company paid Haynes $630,000 after deducting $20,000 she had received from Perrier’s insurance carrier.

Issue(s)- Did the trial court err in granting the defendants’ motion for summary judgment? Do underinsured motorists benefits fall within the common law rule precluding double recovery or do they fall within the common law collateral source rule?

Holding- No. The fundamental principle behind the underinsured motorist insurance is to place the insured in the same position as, but no better than, the insured would have been had the underinsured tortfeasor been fully insured. The P’s argument would allow her to recover MORE than her full damages.

Rule(s)- Double-recovery is precluded by policy. We are trying to place the injured party in the same position he/she would have been had the accident not occurred.

Analysis-

Disposition- Affirmed

Notes-Haynes v. Yale-New Haven Hospital- Plaintiff’s mother died in the emergency room following a car wreck caused by another motorist - P settled a wrongful death claim against the motorist, receiving 20K, the limits of the motorist’s liability

insurance policy. - P then brought a claim against the hospital and the surgeon alleging malpractice. Then D’s moved for summary

judgment on the grounds that plaintiff had already received full compensation of 650k. Motion was granted. - Issue: Did the trial court err in finding that the insurance payment from the mother’s insurer under- In other words, should the underinsured motorist coverage be treaes under the double recovery or the

collateral source rule?- Double recovery rule

o You can only recover one time

7

Page 8: Torts 2 Case Briefs

- Collateral Source Rule:o A D is not entitled to a credit against the damage award to P for those amounts

- The court in this case said that the payments by the underinsured motorist coverage insurer- More courts apply the collateral source rule than the double recovery rule to underinsured motorist insurance - Subrogation:

o The right to claim recoupment from proceeds paid to the P from the tortfeasor or the tortfeasor’s insurer for payments to the P by P’s insurer (or employer’s insurer)

o This may in fact result in the right of the subrogated party to sue the tortfeasor directly or to intervene in a lawsuit brought by the injured party to protect the subrogation rights.

o May be granted by statute (typically the worker’s compensation insurer is subrogated by statute to the injured worker’s claim against 3rd persons), or by contract (typically the health insurer is subrogated by contract)

o Example: If a worker’s comp insurer pays benefits such as medical expenses on behalf of an injured

worker, the worker’s comp insurero Typically if a settlement is involved the WC insurer must be involved in the settlement with the

tortfeasor. o Subrogation is also typical in insurance agreements

- Insurance issues:o Who gets stuck with the loss between the insurers?

- Collateral benefits- received by injured parties from sources other than D, such as medical or car insurance. It is something that the Plaintiff will have paid for.

- If you are only entitled to 100k from D, but you have insurance that will pay 50k. How much can you recover? Collateral benefit says that the insurance doesn’t count toward the 100k that D owes.

- TORT REFORM

WRONGFUL DEATH AND SURVIVAL ACTIONS- Common Law permitted no recovery for wrongful death actions. The explanation was that fell under the felony-

merger rule, premised on the notion that the negligent or intentional killing of another was a felony, punishable by death. Thus there was no defendant to sue. Another explanation was that the law generally does not give one person a cause of action for a tort committed against another.

- Mid-19th century, American jurisdictions adopted statutes permitting actions for “wrongful death.” The statutes specified who could recover (usually spouse, children, sometimes parents and siblings, sometimes all the decedent’s legal heirs), and who could bring suit (usually the person authorized to administer the decedent’s estate, sometimes any of the persons authorized to recover. (Statutorily created with designated beneficiaries)

- Wrongful death action and survival actions are both statutorily created - Beneficiaries (varies from state to state), everything is statute provided

o Surviving Spouse is always includedo Almost all if not all include the children o Parents and Siblings may be able to recover in the absence of spouse or children (generally speaking,

they are last in line)o Sometimes other heirs and collateral relatives (out of the immediate family)o Estate or legatees

- Wrongful death), everything is statute providedo Can recover for:

Loss of support Loss of companionship Loss of guidance

8

Page 9: Torts 2 Case Briefs

Loss of services o Some states allow for grief and mental anguish (but not in most states)o Cause of action exists when the deceased could have brought a claim for his own injuries. When there

has been a tort committed against the deceased for which he could have sought recovery if he had survived, then the wrongful death action would apply.

o Typically the estate rep (the executor of the will or the administrator) must bring the suit. In some states, the beneficiaries can bring suit in their own names.

o If various beneficiaries bring suit, typically all beneficiaries must adjudicate their claims in the same suit. o The statute may specifically provide how the proceeds recovered are to be divided among the

beneficiaries. - Survival Statutes

o The claim that the deceased could have brought on her own behalf and then she dies, but she could have filed before she died, then that claim survives

o Allows the claims of deceased that came into existence before death to proceed. Some states require that the action must be filed before death. Others only require the cause of action to have come into being, but do not require filing before death, as long as it falls within the SOL.

o Claims that arose against the decedent before death may be prosecuted against the estate (through the estate representative).

o Damages would typically include: Post-injury, pre-death pain and suffering of the decedent Loss of earnings from the time of injury until the time of death Medical and incidental expenses Funeral expenses Property damage claims

o A number of states do not permit the survival of defamation claims o Punitive damage claims typically do not survive defendant’s death o Some states do not permit punitive damage awards when the injured person dies before the judgmento Medical and funeral expense claims may depend on who pays them or owes them. Sometimes they are

recoverable in the wrongful death action.o A few states allow the recovery of loss of earning capacity in survival actions (i.e., earning lost because

of the death itself), but subtract personal expenses and support to others (or allow recovery for loss of net savings)

o Some states have statutes that in essence merge both wrongful death and survival claims into one action

o Majority- both actions may be brought together - Punitive damages:

o Purpose: To punish (retribution) To deter (defendant and others)

o Punitive damages may be awarded when the defendant’s conduct may be regarded as willful or wanton, or where the defendant acts with malice or reckless disregard of the safety of others.

o An intentional tort doesn’t necessarily mean there will be punitive damages. I.e. a good faith trespass. You must prove the intent.

o Standard of proof: Preponderance of evidence Clear and convincing evidence (Mississippi standard)

It is not beyond a reasonable doubt Requires more than “it is more likely than not” Must be obvious Between beyond reasonable doubt and more likely than not

9

Page 10: Torts 2 Case Briefs

Must be convincing to the jury, can’t be a “he probably did it” Greater than 51%

o Punitive damages award may be considered as part of the principal case or it may be considered separately after any compensatory damage award is made.

o May state not have caps on punitive damageso Some states require that a significant portion of the punitive damages go to the state rather than the

plaintiff on the theory that since they are acting on behalf of the public, it should benefit the public in the state and not only the plaintiff.

ASPINALL v. MCDONNELL DOUGLAS CORPUnited States Court of Appeal, Ninth Circuit, 1980

625 F.2d 325

Procedural History- Appellant Aspinall, as representative of the estate of Anthony Price, appeals from the district court’s order for summary judgment holding that she and her children are not “heirs” to his estate for the purposes of the California Wrongful Death Act.

Facts- Anthony Price was killed in a plane crash that was manufactured by McDonnell Douglas. At the time of his death, he was unmarried and had no issue. His parents were deceased and he had no collateral heirs. However, he and the appellant lived together for over 4 years and the decedent left his estate to her in his will. Price supported the appellant and her children , but never married Aspinall and never adopted her children.

Issue(s)- Did the district court err in holding that Aspinall and her children were not heirs to the Price estate under the California Wrongful Death Act?

Holding- No. The act requires that in order for Aspinall to have been an heir, she and Price would have had to have been married. In order for her children to have been heirs, Price would have had to have adopted them. Because he neither married Aspinall nor adopted her children, Aspinall and her children cannot be considered heirs under the California Wrongful Death Act.

Rule(s)- Section 377 of the California Wrongful Death Act defined heirs as “those who would have been eligible to inherit from the decedent’s estate had he died intestate”- i.e. having no valid will/last testament before death.

Analysis-

Disposition- Affirmed

Wrongful death – for the survivorship (those suing on behalf of the decedent) to recoverSurvival actions – actions for recovery brought prior to the death of the plaintiff

McDAVID v. UNITED STATESSupreme Court of Appeals of West Virginia, 2003.

Page 353

Procedural History- P sued in federal court, alleging negligence by govt. physicians that caused her husband’s death. She also sought damages for his emotional distress and pain and suffering from the time of negligence until his death, and for his loss of enjoyment of life. Government moved to dismiss the claim for pain and suffering.

10

Page 11: Torts 2 Case Briefs

Certified question given: Whether a decedent’s beneficiaries may recover damages for a decedent’s pain and suffering, incurred between the time of injury and the time of death, where the injuries result in death but the decedent did not institute an action for personal injury prior to his death?

Facts- P’s husband died while being treated at a VA hospital.

Issue(s)- Whether a decedent’s beneficiaries may recover damages for a decedent’s pain and suffering, incurred between the time of injury and the time of death, where the injuries result in death but the decedent did not institute an action for personal injury prior to his death?

Holding- Under the wrongful death act, W.Va.Code 55-7-6, a jury’s verdict may include damages for the decedent’s pain and suffering endured between the time of injury and the time of death, where the injury resulted in death but the decedent did not institute an action for personal injury prior to his or her death.

Rule(s)- 1. “Loss to Survivors” Theory- Majority Rule- Under these statutes, damages generally are to be assessed in accordance with the loss to the decedent’s survivors, with many states limiting recovery to the pecuniary losses of the decedent’s survivors caused by the decedent’s death.

Minority Rule for wrongful death acts- measure damages by the loss to the decedent’s estate. These allow for the estate to recover its losses—such as lost income, funeral expenses, medical expenses, or any other damages related to the decedent’s fatal injury.

Disposition- recovery allowed

Notes:Typically all beneficiaries must adjudicate their claims in the same suit. The statute may specifically provide how to proceeds recovered are to be divided among the beneficiaries.

Note p.355-56

Loses to the Decedent’s Estate- Usually wrongful death recovery gives the survivors only the portion of the decedent’s income that he or she

would have spent on them- the financial support they lost by virtue of the wrongful death- Most survival statutes limit recovery to losses sustained before death, but that can put the tortfeasor whose

victim dies in better position than one whose victim is permanently disabled. The victim who is permanently disabled is entitled to recover for lost earning capacity for the remainder of his or her expected life. When the victim is killed, in most states the tortfeasor has to pay only the portion of the earning that the decedent would have contributed to survivors plus the decedent’s lost earnings from the tort until death

- The tortfeasor escapes liability for (a) earnings the decedent would have spent on himself or herself and (b) earnings the decedent would have saved and thus passed on to his or her estate.

- A few states avoid this situation by construing their survival statutes to permit the estate to recover for the wealth the decedent would have likely accumulated during his or her natural lifetime.

Loss of Consortium in Non-Fatal Cases- Once courts began allowing for the loss of society in wrongful death cases, the question naturally arose whether

family members of seriously injured persons could recover such damages in non-fatal cases. - Most of the modern American cases confronting the issue have eliminated the discrimination by recognizing a

wife’s cause of action for lost consortium. - The majority of jurisdictions confine loss of consortium in non-fatal cases to spouses

11

Page 12: Torts 2 Case Briefs

- Some recent decisions have allowed children’s claims for loss of an injured parent’s consortium and parent’s claims for loss of an injured child’s consortium.

OTHER COMPENSATORY DAMAGE ISSUESInterest- Once a judgment is entered in a personal injury case, the amount normally is treated as a debt owned by the

defendant and interest begins to accrue at a rate that is usually set by statute. - Increasingly, courts have been inclined (or direct by legislatures) to award prejudgment interest on the theory

that the tortfeasor’s obligation to make the victim whole arises when the injury occurs, or at least when the suit is filed.

Taxation- Punitive damages are subject to federal income tax, but compensatory damages are not. - In Rodriguez v. McDonald, the trial court forbade the parties from introducing evidence relating to taxation. The

court of appeals approved that procedure and said most jurisdictions agree that jurors should be told nothing about taxation.

- Supreme Court has held that defendants in Federal Employers Liability Act (FELA) cases are entitled to have the jury told that compensatory damages are not taxed.

Periodic Payments- Problems with lump sum payments:

o Awards for pecuniary losses usually are reduced to present value on the theory that the plaintiff can use the award to make more money by investing it. If the plaintiff chooses to blow the money, the tort system’s objective of making the injured party self-sufficient is frustrated.

o It requires that all future losses be calculated at one moment in time, precluding future adjustments for changes in earnings, medical costs, treatment methods, inflation, and the plaintiff’s condition

- A number of states permit or even require the court to arrange for periodic payment of certain types of judgments (ex- medical malpractice or against governmental entities)

Structured Settlements - Far more common are settlement agreements providing for periodic payments rather than a lump sum. - Parties often negotiate a structured settlement in which the defendant agrees to buy the plaintiff an annuity

that guarantees the plaintiff specified monthly or annual payments for a specified period such as the rest of the plaintiff’s life, a fixed number of years, or until the plaintiff ultimately receives.

Cap on Damages- In more than half of the states, legislatures have imposed caps on compensatory damages- In some states the cap applies only to certain types of cases or certain types of damages - Opponents of caps argue that they make the most seriously injured plaintiffs bear the costs of tortious conduct

while leaving whatever imperfections are thought to require modification of the common law damages rules unaffected in the vast majority of cases where damages do not reach the cap

- Proponents of caps say that they are a simple way of reducing the cost of tort liability and insurance premiums

Alternative Compensation Schemes - Best known alternative is workers’ comp. all states and the federal government have statutes giving employers

immunity from most tort liability for on-the-job injuries or occupational diseases in exchange for their contributions to a fund of insurance plan which pay injured worker benefits in amounts that are administratively determined.

- There are numerous exclusions, e.g., for intentional torts, and attempts by employees to get out of the workers’ comp scheme and into the tort scheme generate numerous litigation

- Another alternative is no fault insurance. The idea is that certain classes of injuries, such as automobile accidents, are compensated by the victim’s own insurance, at least up to a specified amount. The theory is that

12

Page 13: Torts 2 Case Briefs

this is less costly than a system in which the defendant’s insurance pays only after the victim establishes the defendant’s liability under tort law.

- Occasionally Congress creates ad hoc compensation plans to bypass tort liability in specific situations. I.e.- Victim Compensation Fund after 9/11

PUNITIVE DAMAGES

STATE FARM MUTUAL AUTO. INS. CO. v. CAMPBELLSupreme Court of the United States, 2003

538 U.S. 408, 123 S.Ct. 1513Procedural History- A jury determined that Campbell was 100% liable and returned a judgment for $185,849. State Farm eventually paid the entire award, over an 8 year period, but the Utah courts held that State Farm acted in bad faith in failing to promptly settle the suit against the Campbell’s when they knew that Campbell was at fault. The Utah SC upheld an award of $1 million in compensatory damages for the Campbell’s emotional distress. It also upheld an award of $145 million in punitive damages based on the findings that State Farm’s refusal to settle the case against the Campbells was part of a nationwide scheme to meet corporate financial goals by underpaying claims.

Facts- Campbell was involved in a collision with two other individuals. One was killed and the other was permanently disabled. Campbell’s auto insurance company, State Farm, decided to contest Campbell’s liability in the accident and declined settlement offers from the estates of the others injured. Instead, they took the case to trial.

Issue(s)- Did the lower courts err in granting excessive punitive damages in violation of the due process clause of the 14th

Amendment?

Holding- Yes. The state of Utah did not have a legitimate concern in imposing punitive damages to punish a D for unlawful act committed outside of its’ jurisdiction. The court erred in awarding punitive damages to the Campbells to punish and deter conduct that was not related to the Campbells’ harm. A D should be punished for the conduct that harmed the P, not for being an unsavory individual or business. The punitive award of $145 million was neither reasonable nor proportionate to the wrong committed, and it was an irrational and arbitrary deprivation of the property of the defendant.

Rule(s)- The Due Process Clause of the Fourteenth Amendment prohibits the imposition of grossly excessive or arbitrary punishments on a tortfeasor.

- 3 guideposts for courts to consider when reviewing punitive damages (BMW of N America v. Gore):o The degree of reprehensibility of the defendant’s misconducto The disparity between the actual or potential harm suffered by the plaintiff and the punitive damages

awardo The difference between the punitive damages awarded by the jury and the civil penalties authorized or

imposed in comparable cases.

Disposition- Reversed and remanded

Notes-Punitive Damages may be awarded when the D’s conduct may be regarded as willful or wanton, or where the D’s acts with malice or reckless disregard of the safety of others.

13

Page 14: Torts 2 Case Briefs

-Standard of Proof: Either by preponderance of the evidence OR clear and convincing (MS) evidence

-Punitive damage award may be considered as part of the principal case or it may be considered separately after any compensatory damage award is made.-Many states now have damage caps on punitive damages. Some states require that a significant portion of the punitive damages go to the state rather than the plaintiff.

- State Farm v. Mutual Auto. Ins. Co. v. Campbell- Constitutional issue: Due Process Clause (14th Amendment) - State Farm had a duty to settle because they had a chance to settle at the policy amount, which was less than

what the insured was exposed to. In this case, the exposure of Campbell was much greater than the policy limit. - BMW of North America:

o 3 guideposts Degree of reprehensibility Disparity between actual/potential harm to plaintiff and the punitive damage award The disparity between the punitive damage award and the “civil penalties authorized or

imposed in comparable cases”- Factors in considering reprehensibility

o Type of harm- physical/economic mental state (indifference/reckless disregard)o Vulnerability of the victim financially repeated or isolated misconducto Motive- malice, fraud, or accident

- Disparity between actual/potential damages and punitive award”o No bright-line ratio, but court will look unfavorably on awards that exceed a single digit ratio. o Larger the compensatory award, the smaller the ratio that is likely to be awarded

- Differences between civil penalties that are authorized or imposed in similar cases and the amount of punitive awarded by the jury

- Other limits:- State law cannot reach conduct of the defendant that is lawful in another state to punish such conduct. - Punitive damages must relate to relevant conduct - Defendant must have notice that the conduct is subject to punitive damage claims

Scalia and Thomas dissented on the ground tha the const does not constrain the size of punitive damages

14

Page 15: Torts 2 Case Briefs

CHAPTER 9- AFFIRMATIVE DEFENSES BASED ON PLAINTIFF’S CONDUCT

INTRODUCTION- Affirmative defenses- any matter that, is pleaded and proved by the defendant, will defeat or reduce the liability

that plaintiff has otherwise established. The defendant has the burden of proof to show that the plaintiff engaged in some kind of conduct that resulted factually and legally in his/her own injury.

- In bearing the burden of proof, the defendant may rely on any relevant admissible evidence introduced by any party to the action, not just evidence introduced by the defendant.

- Examples of affirmative defenses:o Release (as in a settlement agreement)o Discharge in bankruptcyo Res judicata- the thing has already been adjudicated. The court has already entered a judgment and it

can’t brought again o Estoppel- bared from bringing this claim due to some legal impedimento Arbitration and awardo Fellow servant doctrine (Worker’s Comp)o Duress, fraud, illegality, failure of consideration, accord and satisfaction, SOF, waivero Lacheso Statute of Limitations

- Rule 8(c) of the FRCP requires a defendant relying on an affirmative defense to “affirmatively” plead it, and lists as the affected defenses:

o Accord and satisfaction, arbitration and award, assumption of risk, contributory negligence, discharge in bankruptcy, duress, estoppel, failure of consideration, fraud, illegality, injury by fellow servant, laches, license, payment, release, res judicata, SOF, SOL, waiver, and any other matter constituting an avoidance or affirmative defense.

- In most states, the defendant must plead and prove contributory or comparative negligence and assumption of risk

- Traditional rule was contributory negligence- negligence of the plaintiff in being a factual and legal cause of his or her own injury is a complete bar to plaintiff’s recovery (recovered nothing)

- Courts attempted to mitigate harshness: kind of an all or nothing proposition o Ignoring plaintiff’s negligence or applying a double standard

What was negligent for the D, the same conduct was not deemed to be negligent for the P. The court would say that the duty you owe to yourself is not as important as the duty that you owe to others. (Not very sensible)

o Last clear chance doctrine If the P is negligent in putting herself in a position of peril, and the D is negligent in failing to

perceive that peril and to avoid it, then the D has the last clear chance to avoid the accident, then the P’s negligence is ignored and all of the blame falls on the D.

o Plaintiff’s negligence is slight in comparison to defendant’s o Defendant was grossly negligent or willful and wanton (reckless) o Passive v. active negligence

If the P is passively negligent and D is actively negligent, then D should be responsible for the loss

- All or nothing approach:o Either P or D bore the entire loss, even though both were at fault

- Under common law rules, if the tort is intentional, the P’s negligence does not matter- If there is an intentional tort, the damages might be able to be mitigated (inducement or provocation)

15

Page 16: Torts 2 Case Briefs

- Comparative Negligence o Liability is apportioned or shared according to assessment of fault or responsibility o Mississippi was the first state to adopt the comparative negligence standard (1910)- it was adopted by

statute o Not until the 1960’s did other states begin moving toward a generally applicable comparative negligence

regime for torts o By court adoption or by legislationo Mississippi did so by legislation. o 5 contributory jurisdictions:

Alabama Maryland North Carolina Virginia Washington, D.C.

CONTRIBUTORY AND COMPARATIVE NEGLIGENCE

HOFFMAN v. JONES Supreme Court of Florida, 1973

280 So.2d 431

Procedural History- The trial judge denied P’s request to instruct the jury on comparative negligence. The trial judge gave the standard contributory negligence instruction, and the jury returned a verdict for the defendant. P appealed to the intermediate appellate court, which certified the following question the state supreme court: Whether or not the Court should replace the contributory negligence rule with the principles of comparative negligence?

Facts- P’s husband was killed in a traffic accident that resulted from a combination of his own negligence of the other driver.

Issue(s)- Did the trial court err in denying P’s request to instruct the jury on comparative negligence?

Holding- Yes. A plaintiff in an action for negligence will no longer be denied any recovery because of his contributory negligence. If it appears from the evidence that both P and D were guilty of negligence which was, in some degree, a legal cause of the injury to the P, this does not defeat the P’s recovery entirely. The jury in assessing damages as in the jury’s judgment the negligence of the D caused to the P. In other words, the jury should apportion the negligence of the P and the negligence of the D; then in reaching the amount due the P, the jury should give the P only such amount proportioned with his negligence and the negligence of the D.

Rule(s)-1. To allow a jury to apportion fault as it sees fit between negligent parties whose negligence was part of the legal

and proximate cause of any loss or injury, and2. To apportion the total damages resulting from the loss or injury according to the proportionate fault of each

party Analysis-

Disposition- Remanded to trial court for a decision consistent with this ruling

16

Page 17: Torts 2 Case Briefs

Notes- - Should Florida judicially adopt comparative negligence regime in place of contributory negligence rule?

o Yes… the court did adopt this as the new rule - Contributory negligence is a judicially created rule- Generally regarded as unjust and harsh - Movement has been away from contributory negligence toward comparative negligence - Justifications for contributory negligence are not valid in the modern era- Comparative negligence is more equitable. - Contributory negligence rule is often disregarded by juries. - Efforts by the courts to ameliorate the harshness of the rule are unsatisfactory - Florida adopted “pure” comparative fault

o Plaintiff may recover damages in the amount of the percentage of responsibility of the defendant for the accident after apportioning responsibility between the plaintiff and the defendant (if the plaintiff is also at fault). It doesn’t matter what the % are, as long as they add up to 100%.

- Modified 51% system, and modified 50% system o If the P’s negligence exceeds a certain %, then the D is not required to pay for any damages. o One system says if there is a 50/50- P gets nothing- Must be 50% or less for P to recovero The other says if the P is above 51%, P gets nothing – If P is equally or more negligent than D, then P

does not recover o Majority of states that use modified rules, use the 50% standard.

Look at n.7 p.385, regarding set-off

Jury instructions in modified systems- do you tell the jury about the effect of p’s negligence? –tendency to tell the jury, but maybe half and half

WASSELL v. ADAMSUnited States Court of Appeals, 7th Circuit, 1989.

855 F.2D 849Page 386

Procedural History- Trial Court jury awarded P $850,000 worth of damages, which was reduced to $25,000 since she was ruled to be 97% at fault. P’s attorney moved for JNOV on the grounds that she had either not been negligent as a matter of law or that her negligence was immaterial because the D’s negligence was willful and wanton in their disregard for her safety. Judge denied to motions and P appeals.

Facts- P was a tenant of the Rin-Ric Motel, which was owned by the defendants. This was located in a bad area of town. P was awoken by a knock at her door around 1:00 a.m. A well dressed black man was on the other side of the door. He asked for another woman, and was told that there was nobody by that name there. He then asked for a glass of water to which the P obliged. When she returned from fetching the water, the man was in her room. He proceeded to rape her. Subsequently, P has suffered great emotional distress.

Note- After the date the action was filed, IL adopted a new statute that bars a P from recovery when they are more than 50% negligent in causing the injury for which recovery is sought.

17

Page 18: Torts 2 Case Briefs

Issue(s)- Did the district court judge err in denying P’s request for JNOV? Did the district court judge err in not allowing P’s motion for a new trial?

Holding- No. No rational jury could find that the D’s consciously disregarded a high probability of serious physical harm. Thus, their negligence was not willful or wanton, and must be considered simple negligence.

Rule(s)- The common law refused to compare the P’s and the D’s negligence. The negligent P could recover nothing, unless the D’s culpability was of a higher degree than simple negligence (willful or wanton).-Appellate Courts are not the trier of fact, and they are only authorized to upset the jury’s apportionment if persuaded that the trial judge abused his discretion in determining that the Jury’s verdict was not against the clear weight of the evidence.

Reasoning/Analysis- It is unlikely that a warning would have averted the attack. P testified that she thought the man who had knocked on the door was her fiancé. Thinking this, she would have opened the door no matter how dangerous she believed the neighborhood to be.

Disposition- Affirmed.

Notes-- Appeal from denial of motions for JNOV- What happens to the distinctions at common law involving “willful and wanton” and “gross” negligence?- Do these survive insofar as preventing plaintiff’s negligence from being considered in reduction of her recovery? - What should the effect of conscious disregard of another’s safety be in a comparative negligence system? - How do juries apportion damages between a P and D when both have acted negligently?- Is it really a matter of comparing costs of avoidance?- Is it a matter of declining the degree of departure from the norm? Is it mostly an intuitive matter? - Ameliorative doctrines- losing favor- Intentional torts

o Comparative fault, good or bad- does it depend on the facts? E.g.- mutually agree fight (or in the circumstance of fighting words exchanged) v. Wassell rapist

situation?

FAILURE TO AVOID CONSEQUENCES; FAILURE TO MITIGATE DAMAGES

DARE v. SOBULESupreme Court of Colorado, 1984

674 P.2d 960

Procedural History- Petitioner’s (Dare) brought this action against Sobule to recover damages for the wrongful death of their son. At trial, two witnesses testified that decedent was not wearing a protective helmet at the time of the accident. Petitioners did not object to the introduction of the evidence. The trial court refused to enter the evidence to the jury that failure to wear a helmet while operating a motorcycle is not contributory negligence. The jury found that both the decedent and the defendant were negligent. The decedent was found to be 80% liable and the defendant was 20% liable. Because Colorado is a modified comparative fault state, the plaintiff took nothing and appealed. The Circuit court of appeals affirmed.

Facts- Tracy Dare, decedent, was riding a motorcycle when Sobule, who was driving an automobile made a left turn in front of Dare’s motorcycle. The motorcycle struck the vehicle and Dare was thrown over the car, landed on his head, and died as a result of head injuries.

18

Page 19: Torts 2 Case Briefs

Issue(s)-1. Did the trial court err in refusing to instruct the jury that failure to wear a helmet when riding a motorcycle was

not contributory negligence? 2. Whether the court should impose a standard of conduct for those riding motorcycles?

Holding- 1. Yes. Under the law of comparative negligence in the state of Colorado, evidence of a plaintiff’s failure to wear a

protective helmet while riding a motorcycle is inadmissible to show negligence on the part of the plaintiff or to mitigate damages.

2. No, for three reasons. First, a D should not diminish the consequences of his negligence by the failure of the injured party to anticipate D’s negligence in causing the accident itself. Second, a defense premised on an injured party’s failure to wear a protective helmet would result in a windfall to tortfeasors who pay only partially for the harm their negligence caused. Third, allowing the defense would lead to a battle of experts as to what injuries would have or would not have been avoided had the P been wearing a helmet.

Rule(s)- The General Assembly of Colorado has not mandated the use of protective helmets as a standard of conduct.- Under the law of comparative negligence in the state of Colorado, evidence of a plaintiff’s failure to wear a

protective helmet while riding a motorcycle is inadmissible to show negligence on the part of the plaintiff or to mitigate damages.

Analysis- The improper admission of evidence of failure to wear a protective helmet provides rational explanation for the jury’s findings that the decedent was 80% liable.

Disposition- Reversed and remanded

Notes- - Evidence of failure to wear motorcycle helmet or seatbelt is not admissible to establish contributory negligence

or to reduce the plaintiff’s damages. You don’t need to anticipate the negligence of others.- -Rational behind Dare:

1. Rule avoids windfall to D in allowing him to avoid full consequences of his misconduct2. Avoids battle of the experts (least compelling of the three made here)3. Defendant should not diminish consequences of his negligence by failure of innocent party to

anticipate the defendant’s wrongful conduct - Other arguments

o Absence of legislative mandateo No duty to mitigate before injury occurs

HUTCHINS v. SCHWARTZSupreme Court of Alaska, 1986

724 P.2d 1194

Procedural History- P sued D for $275K in compensatory damages. He filed a motion in limine to exclude evidence of his failure to wear a seatbelt. This request was denied by the trial judge. The judge granted Hutchins’ motion for a directed verdict on the seatbelt issue. The jury was instructed to disregard all evidence relating to Hutchins’ failure to wear a seatbelt. The jury found that Hutchins was 40% liable and Schwartz was 60% liable. The awarded Hutchins $1,937.09 in damages. Hutchins moved for JNOV and/or a new trial. This motion was denied and P’s appealed.

19

Page 20: Torts 2 Case Briefs

Facts- Hutchins was injured in an automobile accident with Schwartz. Hutchins was not wearing his seatbelt at the time of the accident and sustained cuts on his head, bruises on his chest, knee and wrist, and a broken big toe.

Issue(s)- 1. Did the trial court err by admitting evidence of Hutchins’ failure to wear a seatbelt?2. Did the trial court err by denying his motion for JNOV and/or new trial?3. Whether the court should impose a duty upon a person to wear a seatbelt when driving a car equipped with

one?

Holding-1. No. If under the facts and circumstances of the case a reasonably prudent person would have used a seatbelt

and if plaintiff suffered more severe injuries as a result of not wearing a seatbelt, then the jury should be permitted to consider the factor in assessing damages.

2. No. After reviewing the evidence, reasonable minds could differ as to whether Hutchins’ headlights were on or off. Additionally, the jury could have found that Hutchins was traveling too fast for the road conditions. In conclusion, there is an evidentiary basis for the jury’s finding that Hutchins was 40% comparatively negligent.

3. Yes. Automobile accidents are foreseeable. The fact that many motorists do not wear seatbelts may suggest that a failure to wear a seatbelt does not violate a substantial standard of care. However, the fact that a majority of people act in a certain manner does not make that conduct reasonable, especially when that conduct involves an unnecessary risk.

Rule(s)-- Foley v. City of West Allis- In light of the realities of the frequency of automobile accident and the extensive

injuries they cause, the general availability of seatbelts, and the public knowledge that riders and drivers should “buckle up for safety,” those who fail to use available seatbelts should be held responsible for the incremental harm caused by their failure to wear available seatbelts. A plaintiff only recovers damages for injuries caused by defendant and not for those that plaintiff could have prevented by wearing a seatbelt.

- Insurance Co. of North America v. Pasakarnis-In light of the importance of the seatbelt as a safety precaution and the minimal effort required to fasten an available seatbelt, the court concluded that failure to wear one could be a pertinent factor for the jury to consider in determining damages.

Analysis-Disposition- Affirmed Notes-

- This is a minority view. Many states forbid introduction of seatbelt evidence to show comparative fault negligence. In some jurisdictions it can be introduced in products liability, and a few other areas.

- Failure to avoid consequences is generally not required. You are not generally required to anticipate the negligent conduct of another person. Actions that could have been taken by P beforehand. Generally they do not have a negative impact on P’s recovery.

- Failure to mitigate damages refers to what you do after the injury happens. Plaintiff has a duty to act reasonably after the damage is done in order to recover. If he fails to do so, the D may be entitled to reduction of damages. He is not liable for the value of the enhanced injury that is attributable to the P’s failure to mitigate. Use comparative fault to allocate that value.

- Comparative (or contributory) negligence: Pre-accident conduct by the victim that was a cause of the accident and hence all of the injuries or damages. Just because you are in a comparative negligence system, does not mean that you do not talk about the P’s contributory negligence in the accident.

- Notes 2 and 3 on page 401

20

Page 21: Torts 2 Case Briefs

ASSUMPTION OF THE RISK

BENNETT v. HIDDEN VALLEY GOLF AND SKI, INC.United States Court of Appeals, Eighth Circuit, 2003

318 F.3d 868

Procedural History- Bennett’s action against Hidden Valley alleged that Hidden Valley was negligent in the design, maintenance, and staffing of its skiing facilities; in the supervision of its customers “so as to prevent or cure dangers created by such business invitees”; in providing its customers with “unrestricted access to advanced and intermediate ski areas without assessing (their) ski aptitude, ability, or experience”; in permitting obstructions, including tress and jumps, to “exist in the path of skiers at a time when (it)…should have known that such obstructions posed a hazard or risk of injury”; in failing to “warn of dangers and obstructions which it knew or reasonably should have known were present at its facilities and ski slopes”; and in failing “to guard against, barricade, protect, or cushion knowable obstructions in the path of skiers upon its ski slopes.” Hidden Valley denied negligence and raised assumption of the risk as a defense. The case proceeded to a jury trial. At the close of all of the evidence, Bennett moved for a judgment as a matter of law, claiming that Hidden Valley had not established its affirmative defense as assumption of the risk. The DC denied the motion. The jury returned a verdict in favor of Hidden Valley. Bennett appealed.

Facts- Bennett went with two older male friends to Hidden Valley for a midnight ski lesson. She was 16 years old at the time. She had limited experience as a skier. While she was skiing down a slope marked for intermediate difficulty, she fell on a bump in the ski slope. She was thrown about 5 feet forward and hit the ground limp. The bump on the slope was not intentionally created by Hidden Valley, but had formed as skiers and snow boarders cut across the slope and moved the snow. Bennett claims injuries as a result of the accident, including brain damage and a diminished future earning capacity.

Issue(s)- 1. Did the DC err in instructing the jury to find for Hidden Valley if the conditions Bennett encountered “were risks

inherent in the sport of skiing?” 2. Did the DC err in denying Bennett’s motion for judgment as a matter of law because Hidden Valley did not make

out an assumption of risk defense?

Holding- No and no. Under Missouri law, a voluntary skier assumes the risk inherent in or incidental to skiing, regardless of her subjective knowledge of those risks. The proprietor of a ski area has no duty to protect a skier from those risks incidental in skiing. By directing the jury to find for Hidden Valley if it determined that the conditions on the ski slope at the time Bennett was injured were inherent risks of skiing, Instruction 7 fairly and adequately submitted the issue to the jury. The DC therefore did not abuse its discretion by giving this charge.

Rule(s)- Affirmative defense in the State of Missouri:- The defense applies where the parties have voluntarily entered a relationship in which the plaintiff assumes

well-known incidental risks.” - A plaintiff’s consent to assume the risk is implied from the act of electing to participate in the activity and as to

those risks, the defendant has no duty to protect the plaintiff1. A participant in sport accepts those hazards that reasonably inherent in the sport so far as they are obvious and

usually incident to the game2. The patron subjects himself to the dangers necessarily and usually incident to and inherent in the game. This

does not mean that he assumes the risk of being injured by the proprietor’s negligence but that by voluntarily entering into the sport as a spectator but that by voluntarily entering into the sport as a spectator he knowingly accepts the reasonable risks and hazards inherent in and incident to the game.

21

Page 22: Torts 2 Case Briefs

Analysis-

Disposition- Affirmed

Notes- - Assumption of the risk- Three uses of the term:

o Express assumption of the risk When there is a written assumption of the risk in writing by the P, who waives the negligence

claim prior to the injury (ex- amateur drag racing) o Implied primary assumption of the risk

Come about in the context of comparative negligence In the nature of a lack of duty- more like a no duty rule Used to be a complete bar to recovery

o Secondary implied assumption of the risk Most often associated with assumption of the risk A knowing, involuntary and unreasonable knowing of the risk Use to be a complete bar to recovery

- Express assumption of the risko Contractual in nature- an agreement not to hold someone liable for negligence prior to some activity or

occurrence. No duty issue. - Implied Primary Assumption of the Risk

o No duty o Plaintiff assumes any risk inherent in an activity (typically as a participant or spectator in a sport)o Objective test- is the risk inherently dangerous? And is it generally known?

- Secondary Implied Assumption of the Risk o Plaintiff has knowledge and appreciation of the danger posed by defendant’s negligence and voluntarily

chose to expose himself to the danger (most states that use comparative fault have abolished this kind, still applies in AL), subjective and arguably stronger than arguing contributory negligence

- Mississippi has abolished assumption of the risk as a separate defense that acts as a total bar to recovery and treats the plaintiff’s conduct under general principles of comparative negligence, except in products liability cases

o In products liability cases in MS, assumption of the risk is a complete bar to recovery - Note 4

IMPUTED CONTRIBUTORY FAULT

CONTINENTAL AUTO LEASE CORP. v. CAMPBELLCourt of Appeals of New York, 1967.

19 N.Y.2d 350Page 411

Procedural History- Trial Court jury awarded judgment for Continental, even though both drivers were found to be contributory negligent. Appellate Division affirmed.

22

Page 23: Torts 2 Case Briefs

Facts- Continental Auto Lease Corp. rented a car to a gentlemen by the name of Kamman for a 4 day period. During said rental, Kamman was involved in an automobile accident with Ralph B. Shepard. Upon the trial, the jury found both drivers negligent, but returned a verdict for Continental. Shepard has since died, and Campbell is his administrator.

Issue(s)- Did the trial court err in ruling that the negligence of Kamman was not imputable to Continental? Or if it was imputable to Continental, should Continental’s recovery have been barred by contributory negligence.

Holding- No. Continental had no interest in where or when the vehicle was driven and no relationship to Kamman consistent with the inference that it had the right to control in any manner Kamman’s conduct as a driver. Accordingly, Kamman’s negligence should not be imputed to Continental to bar its recovery in this action.

Rule(s)-1. One who is injured by the negligent operation of a motor vehicle should have recourse to a financially responsible defendant.

2. If a car owner’s relationship to the driver of his car is such that a degree of physical control over the driver can reasonably be deemed to exist, then the negligence of the driver can be imputed to bar the owner’s recovery against a negligent 3rd party.

Reasoning/Analysis-

Disposition- Affirmed.

Notes- Imputed contributory negligence

- Similar in concept to vicarious liability - But purpose is different- to restrict liability rather than broaden liability - Imputed contributory negligence is when the plaintiff is held accountable in terms of assessing responsibility for

an injury for the negligence of someone else. That person’s negligence is treated as if it were the plaintiff’s own negligence in terms of its effect on the plaintiff’s recovery against a third person. If P were D, could he be held vicariously liable? If yes, then his claim can be imputed

- If the employer is vicariously liable, when the employer is the P, the employees negligence can be imputed to the employer to offset the damages of a third party

- Most common ways to impute damages (Both Ways Rule) o Employer/ Employeeo Employer/ Independent Contractor o Joint Venture or Enterprise (Partnership in business)

- Imputed damageso Derivative claims- When the plaintiff’s cause of action is derivative- based on an injury to someone else

(consortium, parent’s claim for medical expenses incurred for child (not always), wrongful death and survival, bystander)

- Case Noteso NY 1967:

Is the fault of a bailee of a commercial bailor imputable to the bailor in the bailor’s claim against the bailee’s co-tortfeasor?

o No imputed negligence to commercial bailor for bailee’s negligence in claim by bailor against third person for negligent damage to bailor’s property.

23

Page 24: Torts 2 Case Briefs

WHITE v. LUNDERSupreme Court of Wisconsin, 1975.

66 Wis.2d 563Page 414

Procedural History- Trial Court ruled in favor of defendant, ruling that P/Appellant, Lloyd White, was precluded from recovery because he and his wife were 63% contributory negligent.

Facts- Rosemary White and her husband, Lloyd White, as well as James Lunder were involved in a boating accident. The trial court jury apportioned the causal negligence among the parties to Rosemary White, 30%; Lloyd White, 33%; and Lunder, 37%. The Wisconsin statute for comparative negligence provides that contributory negligence shall not bar recovery if such negligence was not as great as the negligence of the person against whom recovery is sought, but any damages allowed shall be diminished in the proportion to the amount of negligence attributable to the person recovering. The trial court judge combined the negligence apportionment of Plaintiff and his wife (63% total). Accordingly, he was barred from recovery.

Issue(s)- Did the trial court judge err in combining the negligence of both the Plaintiff and his wife for the purposes of comparing negligence with that of the third party for a claim for medical expenses and loss of consortium? Are these claims derivative?

Holding- Yes. These were derivative actions. A workable construction, consistent with the statute, that will allow recovery in derivative actions where the causal negligence of the person against whom recovery is sought is greater than either the husband or wife can be accomplished by reducing the entire award both medical expenses and loss of consortium by the percentage of negligence attributed to the injured spouse; and further reducing the entire award by the percentage of causal negligence attributable to the claiming spouse. By this method the person who was found to be causally negligent in greatest degree cannot escape all liability but his liability is decreased by an amount proportionate to the two other tortfeasors.

Rule(s)- A person should not be precluded for summary judgment, so long as his contribution to the negligence is less than that of the person to which he seeks to recover, when dealing with derivative claims. Additionally, when dealing with multiple parties who are contributory negligent to an accident, the apportionments should not be added together for the purposes of deciding whether recovery should be precluded.

Reasoning/Analysis-

Disposition- Reversed. Award for Plaintiff/Appellant consistent with this opinion.

Notes- - What is the effect of the wife’s negligence on husband’s consortium claim against defendant?- Wisconsin’s early comparative neg standard- if P’s negligence is equal to or > D’s negligence= no recovery - Note 2 after case

24

Page 25: Torts 2 Case Briefs

- Imputed negligence doesn’t apply to claims by persons against others as to whom negligence might be imputed when an “outsider” makes the claim

25

Page 26: Torts 2 Case Briefs

CHAPTER 10- OTHER AFFIRMATIVE DEFENSES

STATUTES OF LIMITATIONS

JOLLY V. ELY LILLY & CO.Supreme Court of California, 1988.

Page 418

Procedural History-

Facts- P was born in 1951 from a mother who ingested DES for the prevention of miscarriage. In 1972, P learned that DES daughters could suffer injuries from DES in utero. She went for a check up and was diagnosed as having adenosis. In 1976, she had an abnormal pap smear and underwent a procedure to remove the abnormal tissue. In 1978, P underwent a complete hysterectomy and a partial vaginectomy in order to remove malignancy. As of 1972, P was aware or at least suspected, that her condition was a result of her mother’s ingestion of DES during pregnancy. Starting in 1972, P attempted to find the manufacturer of the drug. Unfortunately, the doctor who had provided the medication had died, and the pharmacist who filled the prescription did not have records indicating the brand of DES prescribed. There is a 1 year statute of limitations for an action “for injury caused by the wrongful act or neglect of another.”

Issue(s)- Did the trial court err in ruling that the Plaintiff’s claim was barred by the statute of limitations?

Holding- No. The limitations period begins when the P suspects, or should suspect, that she has been wronged. Here, P suspected as much no later than 1978. Because she did not file suit until 1981, he suit, unless otherwise saved, is time-barred.

Rule(s)- Discovery Rule- Provides that the accrual date of a cause of action is delayed until the P is aware of her injury and its negligent cause. A plaintiff is held to her actual knowledge as well as knowledge that could reasonably be discoverable through investigation of sources open to her.

Once the P has a suspicion of wrongdoing, and therefore an incentive to sue, she must decide whether to file suit or sit on her rights. So long as a suspicion exists, it is clear that the P must go find the facts; she cannot wait for the facts to find her.

Doe Party Rule- Pursuant to Cal. Code of Civ Pro. Section 474, P could have filed suit against a “Doe Party.” In these actions, from the time such a complaint is filed, the P has 3 years to identify and serve the defendant.

Reasoning/Analysis-

Disposition- Affirmed.

Notes-

- Statutes that prescribe the period of time in which an action must be brought or which will then be time-barred

26

Page 27: Torts 2 Case Briefs

- Affirmative defense. Defendant must plead and prove the facts that would establish the existence of a time bar.- Awareness or knowledge is both actual and constructive- Constructive knowledge is knowledge that is reasonably discoverable through investigation of sources available

to plaintiff - How most states start the SOL:

o Last point in time of continuing torto Time of injury

- Jolly v. Eli o Discovery rule- accrual date of a cause of action is delayed until the plaintiff is aware of her injury and

its negligent cause o SOL in this case was when she has enough facts that would give her reasonable suspicion there was a

causal connection between her injury and someone else’s misconduct or negligence. She had been suspicious since at least 1972 of the DES product. Her SOL had at least run out by 1972.

o SOL avoids stale claims o Ignorance of legal significance of facts- statute continues to run o SOL begins to run when the plaintiff has a suspicion or should have a suspicion that someone has injured

her by wrongdoingo Note 4- 419 (footnote in case)

D’s argue that the statute should commence when the P knows of her injury and its factual cause.

o Once P has knowledge sufficient to excite suspicion of inquiry, the statute begins to run. The P must take steps necessary to preserve her right to a remedy by investigating and bringing suit within the statutory period. (Discovery Rule) -----More commonly applied version

o Policy reasons for SOL: Protect D from defending stale claims Loss of evidence Inability to plan for future contingencies Plaintiffs should diligently pursue their claims.

o Effect of change in the law after the statute has otherwise run? None. The claim is time-barred. A change in the law does not revive a claim that is time barred.

o Policy reasons for not reopening time-barred cases Case management problems for court Need for finality for parties, particularly defendants Stale claims

o Doe Party Complaintso Discovery Rule

Not applied in every state and limited to specific kinds of cases in other states Often SOL’s do not run against minors or others with mental incapacity.

o Note 2 When the tort occurs, look at the statutes

o Note 3 Facts establishing wrongdoing Or facts establishing cause of injury whether or not “wrongdoing” is suspected or known

o Ps may have other theories available to them.o Ps are free to seek change in the common law when they believe the law to be unjust or inadequate.

FELTMEIER v. FELTMEIERSupreme Court of Illinois, 2003

798 N.E.2d 75

27

Page 28: Torts 2 Case Briefs

Procedural History- Trial court denied Robert’s motion but certified all three issues for interlocutory appeal. The appellate court affirmed and Robert appealed. On the first issue, the SC held that the conduct alleged was extreme and a there was a valid claim of IIED.

Facts- Plaintiff, Lynn Feltmeier ,and Defendant, Robert Feltmeier, were married in 1986 and divorced in 1997. In 199, Lynn sued Robert for IIED. She alleged that he engaged in a pattern of emotion and physical abuse which began shortly after the marriage and continued through and after the divorce. Lynn alleged that Robert hit her, kicked her, prevented her from leaving the house, threw things at her, abused her verbally, and, after the divorce, stalked her. Robert moved to dismiss on the ground that the complaint failed to state a claim and that the claim was barred by the SOL.

Issue(s)- Did the appellate court err in holding that the conduct was a continuous tort and the SOL of limitation did not being to toll until after the divorce was final?

Holding- No. The two year SOL for this action began to run in August of 1999, because Lynn’s complaint includes allegations of tortious behavior by Robert occurring as late as that month. Applying the continuing tort rule to the instant case, Lynn’s complaint, filed August 25, 1999, was clearly timely and her claims based on conduct prior to August 25, 1997, are not barred by the applicable statute of limitations.

Rule(s)-- Continuing tort rule- where a tort involves a continuing or repeated injury, the limitations period does not begin

to run until the date of the last injury or the date the tortious acts cease - General SOL rule- where there is a single overt act from which subsequent damages may flow, the statute

begins to run on the date the defendant invaded the plaintiff’s interest and inflicted injury, and this is so despite the continuing nature of the injury

- Pavlik v. Kornhaber holding- “Illinois courts have said that in many contexts, including employment, repetition of the behavior may be a critical factor in raising offensive acts to actionably outrageous ones. It may be the pattern, course and accumulation of acts that make the conduct sufficiently extreme to be actionable, whereas one instance of such behavior might not be. It would be logically inconsistent to say that each act must be independently actionable while at the same time asserting that often it is the cumulative nature of the acts that give rise to IIED. Likewise, we cannot say that cumulative continuous acts may be required to constitute the tort but that prescription runs from the date of the first act. Because it is impossible to pinpoint the specific moment when enough conduct has occurred to become actionable, the termination of the conduct provides the most sensible place to being the running of the prescriptive period.

Analysis-

Disposition- Affirmed

Notes-Continuing tort:

o Effect on running of SOL?o When the conduct of a continuing nature stops, that’s when the SOL starts running o Problem- what conduct constitutes a continuous tort?

- Continuing tort- a tort involving continuing ot repeated injury. Generally involves a pattern of conduct as opposed to one act

- SOL does not begin to run until the date of the last injury or the date the tortious acts cease. - Distinguishing continuing ill effects from continuing unlawful conduct - Single acts v. repeated conduct that is seen as a continuous whole- Pattern of conduct that as a whole can be seen as extreme and outrageous conduct necessary to establish IIED. - - Med Mal or Legal Mal- Note 2

28

Page 29: Torts 2 Case Briefs

STATUTES OF REPOSE

BRADWAY v. AMERICAN NATIONAL RED CROSSUnited States Court of Appeals, Eleventh Circuit, 1993

992 F.2d 298

Procedural History- Plaintiff and her husband filed a complaint alleging that P contracted AIDS during her 1983 blood transfusion. P’s sought compensatory damages, contending that the Red Cross was negligent in screening blood donors and in testing blood samples for the presence of HIV. The Red Cross moved the DC to dismiss the action as barred by Georgia’s SOL and ultimate repose for med mal suits. The DC concluded that under GA law, “an action against a blood bank for the negligent collection and supply of human blood is an action for med mal,” and dismissed the case as barred by GA’s statute of repose. P’s argue that the action did not accrue until the wrong was completed, i.e., when P became infected, and that “it is a jury question as to when P became infected with the AIDS virus.”

Facts- Plaintiff underwent reconstructive surgery for facial birth defects. She received 2 units of whole blood by transfusion after the surgery. The hospital obtained the blood from an American National Red Cross blood bank. The Red Cross had no direct connection with the Plaintiff.

Issue(s)- Did the DC err in classifying the case as a medical malpractice suit barred by GA’s statute of repose rather than classifying it as an ordinary negligence case?

Holding- No.

Rule(s)-- O.C.G.A. § 9-3-70- The term “action for medical malpractice,” means any claim for damages resulting from the

death of injury to any person arising out of:o Health, medical, dental, or surgical service, diagnosis, prescription, treatment, or care rendered by a

person authorized by law to perform such service or by any person acting under the supervision and control for the lawfully authorized person; or

o Care or service rendered by any public or private hospital, nursing home, clinic, hospital authority, facility, or institution, or by any officer, agent, or employee thereof acting within the scope of his employment

- O.C.G.A. § 9-3-71- o Except as otherwise provided in this article, an action for medical malpractice shall be brought within

two years after the date on which an injury or death arising from a negligent or wrongful act or omission occurred.

o Subject to the discovery rule, in no event may an action for medical malpractice be brought more than 5 years after the date on which the negligent or wrongful act or omission occurred.

- SOL- governs the time within which legal proceedings must be commenced after the cause of action accrues. - Statutes of Repose- limits the time within which an action may be brought and is not related to the accrual of

any cause of action. The injuries need not have occurred, much less have been discovered.

Analysis- The plain language of the repose statute indicated that the period begins on the date on which the negligent or wrongful act or omission occurred.

Disposition- Affirmed

29

Page 30: Torts 2 Case Briefs

Notes- - Statutes of Repose- limits time within which action must be brought without regard to when the cause of action

accrued or whether it has yet accrued. - Different from an SOL-

o SOL o SOR- cause of action might have no accrued- runs from the act, which might be wrongful which might

not have triggered any results yet. - Thus, lack of knowledge of the existence of the cause of action does not toll a statute of repose. - In this case, the wrongful act was the blood transfusion… the SOR was triggered at that date, although she did

not test positive for HIV/AIDS for another 5 years. - Filing a case of action against a blood bank is a med mal case, and is barred by 2 years. In addition, the SOR bars

the claim because it gives 5 years from the date of the act. - Examples of types of cases in which legislatures have enacted statutes of repose:

o Medical malpracticeo Architects and building contractorso Products liability

- Negligence involved in the collection and supplying of blood gives rise to a med mal action- SOR began running on the date that the negligent act or omission occurred, not when the negligence was

discovered by plaintiff or when injury occurred. - GA statute bars claims for med mal unless brought within 2 years of the negligent or wrongful act or omission.

(ordinary SOL)- But in no event may such a claim be brought in more than 5 years after the date on which the negligent or

wrongful act or omission occurred - Note 3-

EXPRESS ASSUMPTION OF RISK

HOLZER v. DAKOTA SPEEDWAY, INC. Supreme Court of South Dakota, 2000

610 N.W.2d 787

Procedural History- This case arose from a personal injury accident, when a race car’s wheel detached and hit and injured the plaintiff. The circuit court granted D’s motion for summary judgment based on a pre-accident released signed by P.

Facts- Before entering the pit area of the racetrack (to serve on the pit crew for one of the racers), Holzer was requested by Speedway to sign as “Release and Waiver of Liability, Assumption of Risk and Indemnity Agreement.” All individuals wishing to enter the pit area were required to sign the release form. This document provided that the signees covenant not to sue the track owners, insurers and discharge them from any and all liability. Signing the release was a condition to being allowed into any restricted area such as the pit, and applied to anyone competing, officiating, observing, working for, or participating in races at the speedway. At the bottom of the release, printed by each signature line was the heading “I HAVE READ THIS RELEASE.” P signed and submitted it. He had done this twice before as well.

Issue(s)- Did the lower court err in granting D’s motion for summary judgment based on a pre-accident release signed by the plaintiff?

Holding- No. The release in this matter does not involve a matter of public policy, but rather a private agreement between individuals. P participated in Speedway races as a form of recreation- he was a volunteer and was therefore not

30

Page 31: Torts 2 Case Briefs

compelled in any way to enter the Speedway racing pit area. In addition, there was no evidence in the record that P was denied the opportunity to step out of line at any time and read the release. He was not forced to sign it. If he had problems reading/understanding the release (as his father indicated), he should have asked someone to read and or explain it to him. It is his own fault that he didn’t read it… READ SHIT BEFORE YOU SIGN IT!!!

Rule(s)-- To be valid, a release must be fairly and knowingly made- One who accepts a contract is conclusively presumed to know its contents and to assent to them, in the absence

of fraud, misrepresentation, or other wrongful act by another contracting party Analysis-

Disposition- Affirmed

Notes-- Express Assumption of the Risk Exculpatory Clauses- Explicit, Clear, conspicuous (understandable language) - May not disclaim willful negligence or intentional wrongdoing - May not disclaim liability when a matter of public interest is involved- May not disclaim liability for utilities or quasi-public entities supplying essential services- Meaningful opportunity to be apprised of the waiver- Fairly and voluntarily made - But signatures are not mere ornaments to contracts- See note 1

o Some states choose to limit the ability of parties to waive personal injury claims in some situations - Commercial parties

o Relative bargaining strength- Consumers

o Not equal bargaining power- adhesionary situation - UCC warranty provisions

o Opportunities were parties may waive warranty options concerning negligence and or product liability- Notice of claims

o Sometimes, before filing suit against a state or local govt notice must be provided within a specified period following the injury as a condition of the right to sue.

- Types of Comparative Negligence Regimes:o Pure

Ms LA FL Neglience od P is never a bar to recovery

o Modified 50%

P may recover as long as the P’s negligence is less than that of the D P must be less than 50% negligent TN GA AR

51%

31

Page 32: Torts 2 Case Briefs

P may recover only if the P’s negligence is equal to or less than D’s negligence. As long as P is less than 51% negligent, she will recover.

CHAPTER 11 – COMPARATIVE RESPONSIBILITY IN MULTIPARTY LITIGATION

CONTRIBUTION AND INDEMNITY

BROCHNER v. WESTERN INSURANCE COMPANYSupreme Court of Colorado, 1986

724 P.2d 1293

Procedural History- Trial Court, in an indemnity action to recover monies paid in a settlement to Ms. Cortez, awarded $150,000 to Western Insurance Company for indemnity as well as $10,000 for expenses incurred as a result of the lawsuit. Appellate Division affirmed.

Facts- Dr. Brochner, who was granted staff privileges by the Community Health Association to perform craniotomies at Boulder Community Hospital, was sanctioned by the hospital several times. The Hospital required that Dr. Brochner to acquire consultations before performing craniotomies, as it appeared as though he was performing unneeded surgeries. In November of 1968, Brochner performed a craniotomy on Esther Cortez which resulted in injury to Cortez. Cortez brought an action against the hospital and Dr. Brochner. The two claims were subrogated. The Hospital settled the claim for $150,000. Western sought indemnity from Dr. Brochner on the grounds that his negligence was the primary cause of Cortez’s injury and the Hospital’s negligence was passive and secondary. The trial court awarded Western their requests.

Issue(s)- Did the trial court and appellate division err in ruling that Western Insurance Company was entitled to receive the full amount of the settlement as the Hospital’s negligence had only been secondary and passive to that of Dr. Brochner?

Holding- Yes. A tortfeasor may no longer be unfairly forced to pay all or a disproportionate share of damages suffered by an injured party as the result of negligent conduct by two or more joint tortfeasors. Because both Brochner and the hospital settled with Cortez, neither is entitled to contribution from the other.

Rule(s)- Joint tortfeasors are now subject to contribution among themselves based upon their relative degrees of fault. This new provision extends the principle that liability for negligence should be based on proportionate fault.

Analysis-- As a joint tortfeasor, the hospital has no right to seek indemnity from Brochner; its sole remedy lies in

contribution pursuant to the terms of the act which provides:o When a release or a covenant not to sue or not to enforce judgment is given in good faith to one or two

or more persons liable in tort for the same injury or the same wrongful death: It discharges the tortfeasor to whom it is given from all liability for contribution to any other

tortfeasor. Disposition- Because both Brochner and the hospital settled with Cortex, neither is entitled to contribution from the other

Notes- - Contribution and Indemnity

o Indemnity- the right to full reimbursement for damages paid by one party from another

32

Page 33: Torts 2 Case Briefs

Examples Vicarious liability- employer/employee Contractual indemnity Legal indemnity- seller-manufacturer

o Contribution- the right to obtain a partial reimbursement of damages paid by one party against another The reimbursement is by apportionment between the party who has paid and another party

who has some portion of liability Many jurisdictions with comparative fault apportion by degree of responsibility for the injury

- Abolishing Joint and Several Liability:o A significant number of states have abolished joint and several liability, so that there is no need for

contribution. Each party bears his or her own share of the responsibility.o MS and LA- no longer have joint and several liability for negligence based actionso However, joint and several liability has been retained for acts committed with “specific and wrongful

intent” or for conscious and deliberate wrongs.o Contribution is retained for cases involving such concerts of action

- Bochner v. Western Insurance Co.o A negligent party defendant may not recover full indemnity from another negligent party (joint

tortfeasor), but may only seek contribution in proportion to the fault of the joint tortfeasor. o Under existing CO law in effect before legislative abolishment of joint and several liabilityo The court abolished the “active/passive negligence” distinction for purposes of indemnity because of the

adoption of comparative fault o A good faith settlement of the plaintiff’s claim by one tortfeasor discharges that tortfeasor from all

liability and bars a claim from a joint tortfeasor for contribution

JOINT AND SEVERAL LIABILITY

KAEO v. DAVISSupreme Court of Hawaii, 1986

719 P.2d 387

Procedural History- Kido’s guardian sued Davis and the City of Honolulu. The City’s alleged negligence was failure to maintain the road in a safe condition. The jury found that Davis was 99% negligent and the City was 1% negligent. It also found that P suffered $725k in damages. A judgment holding the City and Davis jointly and severally liable for the damages was entered. City appealed.

Facts- Kido and Davis were drinking at a bar, and then went for a drive. Davis, who had been drinking, was unable to keep the car on the winding road and hit a utility pole. Kido was severely injured.

Issue(s)- Did the trial court err in excluding all evidence of Davis’ alcohol consumption? Did the trial court err in refusing to give a jury instruction “that would have apprised the jury of the possible legal consequences of its verdict on the negligence attributable to each putative joint tortfeasor?”

Holding- Yes. The trial court erred in refusing to admit evidence of Davis’ alcohol consumption. The trial court should have informed the jury of the possible legal consequence of a verdict apportioning negligence among the joint tortfeasors

Rule(s)- The trial court, if requested and when appropriate, should inform the jury of the possible legal consequence of a verdict apportioning negligence among joint tortfeasors.

33

Page 34: Torts 2 Case Briefs

Analysis- Wisconsin uses the Blindfold Rule, whereby the jury is not instructed on the effect that their verdict will have as far as which defendant may ultimately be held responsible for the entire sum. It is not the function of the jury in a case between private parties on the determination of comparative negligence to be influenced by sympathy for either party, nor should it attempt to manipulate the apportionment of negligence to achieve a result that may seem socially desirable.

Disposition- Reversed and remanded.

Notes- Joint and Several Liability - A jury should be informed, when requested by a party, of the legal effect of apportionment of liability among

joint tortfeasors

BROWN v. KEILL Supreme Court of Kansas, 1978

580 P.2d 867

Procedural History- This is an appeal from a judgment for damage to P’s car resulting in a two car accident. The trial court found: (1) that P was guilty of no negligence; (2) the son was responsible for 90% of the causal negligence; (3) the D was responsible for 10% of the causal negligence; (4) P sustained total damages in the amount of $5,423.00; and (5) P was entitled to recover $542.30 in damages from D. P appealed

Facts- P, Brown, allowed his son to drive his Jag, and got into a car accident with D, Keill. The reasonable cost of repair was $5,423.00. D settled her claim against the son out of court. P then sued to recover his property loss. D did not seek to have the son joined as an additional party, and did not file a cross or counter claim against him.

Issue(s)- (1)Did the trial court err in refusing to apply the rule of joint and several liability of joint tortfeasors in the state of Kansas under a comparative negligence statute? (2) Additionally, is the causal negligence or fault of all parties to a collision or occurrence giving rise to P’s claim in a comparative negligence action to be considered even though one of said parties is not served with process or joined as a formal party to the action?

Holding- (1)No Trial court did not err. The concept of joint and several liability between joint tortfeasors previously existing in Kansas no longer applies in comparative negligence actions, due to K.S.A. 60-258(a). Yes, Although D and P both had an opportunity to bring in the third party by way of motion, they did not take advantage of this. There is nothing in the particular statute which specifically requires P to bring his action or file a claim against any particular person or group of persons.

Rules(s)-- Absent evidence of a joint venture, agency or circumstances giving rise to vicarious liability, the negligence of a

bailee of a vehicle is not imputable to the bailor in an action by the bailor against a third party for damage to the bailed vehicle.

- The purpose and legislative intent of K.S.A. 60-258a was to impose individual liability for damages based on the proportionate fault of all parties to the occurrence which gave rise to the injuries and damages even though one or more parties cannot be joined formally as a litigant or be held responsible for his or her proportionate fault.

Analysis-

34

Page 35: Torts 2 Case Briefs

Disposition- Affirmed

Notes- - Absent joint venture, agency, or other special circumstances, bailor is not liable for torts of bailee.- Does KA retain joint and several liability?

o Noo Note p.444 – rule of statutory interpretation (construction, purpose, pari material, etc.)

- Is the negligence of a non-party joint tortfeasor considered in the allocation of responsibility for the plaintiff’s injuries?

o Yes. “Any party” – does this mean parties to the litigation or participants to the tort…- Effect of joint and several liability:

o Each D is liable for the full amount of damages apportioned to the D’s, regardless of the degree of fault of each individual D

o Insolvency- who bears the risk of the insolvent joint tortfeasor? The other remaining solvent Ds will absorb an insolvent party.

VARELA v. AMERICAN PETROFINA COMPANY OF TEXAS, INCSupreme Court of Texas, 1983

658 S.W.2d 561

Procedural History- This is an appeal from the third-party negligence action brought by an employee covered by workers’ comp. Judgment was rendered for the amount of damages found by the jury reduced by the proportion of negligence of both the employee and the employer. The court of appeals affirmed

Facts- Varela was employed by Hydrocarbon, who was performing a “turnaround” on a fluid catalytic cracking unit owned and operated by Petrofina. During the course of performing the job, Varela was injured due to a premises defect. After settling his workers’ comp claim, Valera sued Petrofina for the damages resulting from his injuries. The jury apportioned the negligence of the parties as follows: Varela 15%, Hydrocarbon 42%, and Petrofina 43%. The jury further found damages in the amount of $606,800. The trial court rendered judgment for 43% of the total damages.

Issue(s)- Did the trial court err in considering the negligence of the employer in a third-party negligence action brought by an employee arising out of an accidental injury covered under workers’ compensation insurance?

Holding- Yes. The injured employee may seek recovery from a 3rd party whose negligence contributed to the injury. In the event of recovery, the negligent 3rd party is barred from seeking contribution or indemnity from the employer, and the compensation carrier is entitled to reimbursement for all compensation and medical expenses paid.

-Since the Work Comp Act precluded P from recovering from the employer for common law negligence, Petrofina has no claim for contribution from employer.

Rule(s)- Where the D’s negligence is equal to or greater than that of the employee, the employee shall recover the total amount of damages as found by the jury diminished only in proportion to the amount of the negligence attributed to the employee.

Analysis-

Disposition- Reversed and remanded. Judgment is rendered for Varela in the amount of $515,780 plus interest from the date of judgment.

35

Page 36: Torts 2 Case Briefs

Notes- - Jurisdictions that have abolished joint and several liability have retained it in intentional torts - Vicarious liability is different from Joint and Several liability. Employer is responsible for 100% of damages for

employee acting in nature and scope of employment. Abolition of joint and several liability has nothing to do with vicarious liability.

- Whose fault should be considered in apportioning responsibility?o Non partieso Immune persons

Employers under workers’ comp Governmental immunities Family immunities

o Phantom defendants (non-parties that are alleged to be liability in some capacity)- May a jury consider the negligence of and assign - If not how is fault otherwise attributable to the employer allocated?

o To the non-employer tortfeasor - Note 1- Note 2- Note 3- MS by recent legislature allows employer’s negligence to be considered in allocating percentage of responsibility

for each party involved in cause the injury- MS allows consideration of fault of any actor who had a causal role

Partial Settlements

MCDERMOTT, INC. v. AMCLYDE Supreme Court of the United States, 1994

511 U.S. 202

Procedural History- Jury found damages of 2.1 million. Responsibility was allocated as follows: 32% AmClyde, 38% River Don, and 30% jointly to McDermott and the sling defendants. Court denied motion to reduce the judgment by the $1 million dollar settlement, and entered judgment against AmClyde for $672,000 and against River Don $798,000. Court of Appeals held that a contractual provision precluded any recovery against AmClyde and that the trial judge improperly denied a pro tanto settlement agreement. The Appellate Court reversed the judgment award and reduced it as follows: First, it reduced the 30% negligence on the part of McDermott. This came to $1.47 million. Next, it deducted the 1 million received in settlement to reach $470,000.

Facts- McDermott, petitioner, bought a 5,000 ton crane from AmClyde. When Petitioner first used the crane to attempt to move an oil and gas production platform, a prong off the crane’s main hook broke, causing massive damage to the deck and to the crane itself. The malfunction may have been caused by petitioner’s negligent operation of the crane, by AmClyde’s faulty design or construction, by a defect in the hook supplied by River Don, or by one or more of 3 other companies that supplied the supporting steel slings. On the eve of trial, Petitioner entered into a $1 million settlement dismissing the “sling defendants.” They released them from all liability for either deck or crane damages, and agreed to indemnify them against any contribution action. McDermott assumed any liability the slings may have had in causing the accident.

Issue(s)- Should the liability of the non-settling defendants be calculated with reference to the jury’s allocation of proportionate responsibility? Or should the Court give the non-settling Ds a credit for the dollar amount of the settlement?

36

Page 37: Torts 2 Case Briefs

Holding- The proportionate share rule announced in this opinion applies when there has been a settlement. In such cases, the P’s recovery against the settling D has been limited not by outside forces, but by its own agreement to settle. There is no reason to allocate any shortfall to the other defendants, who were not parties to the settlement. Just as the other defendants are not entitled to a reduction in liability when P negotiates a generous settlement, they are not required to shoulder disproportionate liability when the P negotiates a meager one.

Rule(s)- It is generally agreed that when a P settles with one of several joint tortfeasors, the non-settling D’s are entitled to a credit for that settlement.

3 alternatives outline by the American Law Institute:1. Pro Tanto set off with right to contribution against the settling D-The money paid extinguishes any claim that the injured party has against the party released and the amount of his remaining claim against the other tortfeasor is reached by crediting the amount received; but the transaction does not affect a claim for contribution by another tortfeasor who has paid more than his equitable share of the obligation.

-This discourages settlement, because settlement can only disadvantage the settling defendant.

2. Pro tanto rule without contribution-The money paid extinguishes both any claims on the part of the injured party and any claim for contribution by another tortfeasor who has paid more than his equitable share of the obligation and seeks contribution. As in Alternative #1, the amount of the injured party’s claim against the other tortfeasors is calculated by subtracting the amount of the settlement from the P’s damages.

-The pro tanto approach, even when supplemented with “good-faith hearings,” is likely to lead to inequitable apportionments of liability.

-By disadvantaging the party that spurns settlement offers, the pro tanto rule puts pressure on all defendants to settle. The additional incentive provided by the pro tanto rule comes at too high a price in unfairness.

3. Settling tortfeasor and the proportionate share approach-The money paid extinguishes any claim that the injured party has against the released tortfeasor and also diminishes the claim that the injured party has against other tortfeasors by the amount of the equitable share of the obligation of the released tortfeasor.

-The first 2 alternatives involve the kind of “pro tanto” credit that respondents urge the Court to adopt. The only difference is the recognition of a right of contribution against a settling defendant in the first but not the second. The third alternative involves a credit for the settling defendants’ “proportionate share” of responsibility for the total obligation. The Court uses this approach here.

One Satisfaction Rule- This is no longer good law. Historically, this rule barred a P from litigating against another joint tortfeasor, if he had settled with and released another.

Analysis- The proportionate share rule announced in this opinion applies when there has been a settlement. In such cases, the P’s recovery against the settling D has been limited not by outside forces, but by its own agreement to settle. There is no reason to allocate any shortfall to the other defendants, who were not parties to the settlement. Just as the other defendants are not entitled to a reduction in liability when P negotiates a generous settlement, they are not required to shoulder disproportionate liability when the P negotiates a meager one.

Disposition- Reversed court of appeals decision and remanded

Notes-- What happens if joint and several liability and a D settles for more than his fair share?

o Did the release include or exclude the other D’? If settlement released other D’s also, then contribution allowed (Restatement 3d).

Common law rule- a release to any of the D’s- you release all of them

37

Page 38: Torts 2 Case Briefs

Modified- if you release A, but reserve the right to sue B and C, then you can release A and still sue B and C

Typically the release is a release of the party of who is on the release only… everyone else is still liable if you have a claim against them.

- In most jurisdictions the settling D who pays (at least in good faith) less than what turns out to be his fair share is completely released and no claim for contribution may be made against the settling D.

- Partial Settlementso Is there joint and several liability?

If not, then settlement by on D will not affect the liability of a joint tortfeasor. The nonsettling defendant would pay in the amount that However, where there is joint and several

Several methods have been used:o Pro rata-(equal division among the D’s without regard to % of responsibility for

the injury)o Pro tanto- a dollar for dollar reduction allowed against the award o Proportionate: more modern trend- the award is reduced by the % of

responsibility attributed to the settling defendant - McDermott

o Issue: In an admiralty case, should the amount of settlement with settling D’s who pays more than its

fair share of the damages o Holding

Pro-tanto Proportionate- Court goes with this approach. P’s claim against the settling D is released and the

claim

APPLICATION OF COMPARATIVE RESPONSIBILITY TO CAUSES OF ACTION OTHER THAN NEGLIGENCE

BONPUA v. FAGANSuperior Court of NJ, Appellate Division, 1992

Page 461

Procedural History- Trial Court partially granted P’s Motion for Summary judgment on the grounds that D’s conviction for aggravated assault conclusively established that he committed an intentional tort upon P, and that under the Comparative Negligence Act, any negligence or other fault of P would not bar or reduce his claim. D appeals.

Facts- P and D were involved in a physical altercation in the parking lot behind Cammarano’s Bar in Long Beach on March 26, 1987. This fight stemmed from an altercation in the bar in which P called D a “faggot.” D claims that he walked over to P’s car and P began hitting him, and he was forced to defend himself. Both P and D have suffered injuries. D was convicted of aggravated assault and sentenced to a 7 year prison sentence.

Issue(s)- Did the trial court err in partially granting P’s motion for summary judgment, striking defendant’s affirmative defense of comparative negligence?

38

Page 39: Torts 2 Case Briefs

Holding- Yes. The Comparative Negligence Act requires the tries of fact to apportion the fault of all culpable parties, regardless of whether their conduct was negligent or intentional. Defendant’s criminal conviction does not preclude him from relying upon the Comparative Negligence Act as a defense to P’s claim.

Rule(s)- The Comparative Negligence Act requires the tries of fact to apportion the fault of all culpable parties, regardless of whether their conduct was negligent or intentional.

-A criminal conviction does not place a limitation on the Act

Reasoning/Analysis- The criminal jury could have returned a guilty verdict even though it believed P initiated the altercation if it found that D responded to P’s verbal abuse by assaulting him or used excessive force in repelling P’s aggression. Thus, a finding that both P and D were negligent in causing the injury to P would not be inconsistent with the criminal jury’s finding.

Disposition- Reverse and remanded.

Notes- Apportionment of fault when fault claim is other than negligence - Should comparative fault be used in allocation of fault involving intentional torts?- Should comparative fault be used in allocation of responsibility involving strict liability claims (esp. in products)?

Tendency is for courts to use comparative fault in reducing the claim against the strictly liable party. - Cases involving a mis of intentional torts and negligence - Apportionment by causation and fault in the same case. Mostly unresolved by the courts (Section F. on 466)

39

Page 40: Torts 2 Case Briefs

CHAPTER 12 - IMMUNITIES

A. GOVERNMENT IMMUNITY

1. STATE AND LOCAL GOVERNMENT

HICKS v. STATESupreme Court of New Mexico, 1975

Page 470

Procedural History- This is an appeal from an order of the DC dismissing P’s complaint on the basis of sovereign immunity. Suit was originally brought to recover damages for the wrongful death of P’s wife and daughter for the negligence of the State Highway Dept., when a school bus collided with a cattle truck on a bridge built and maintained by the State Highway Dept. D filed a MTD which was granted by the DC. The DC stated that the doctrine of sovereign immunity was a long-standing common law principle which could now be changed only by legislation.

Facts-

Issue(s)- Did the DC err in granting D’s MTD based on the common law principle of sovereign immunity?

Holding- Yes. The doctrine of sovereign immunity has always been a judicial creation without statutory codification. Just because a court made rule has been in effect for many years does not render it invulnerable to judicial attack once it reaches a point of obsolescence. Sovereign immunity may no longer be interposed as a defense by the state, or any of its political subdivisions, in tort actions. In addition, its continuance is causing a great degree of injustice.

Rule(s)- Common law sovereign immunity may no longer be interposed as a defense by the state, or any of its political subdivisions, in tort actions. It can no longer be justified by existing circumstances and has long been devoid of any valid justification.

- Ayala v. Philie Board of Pub. Education- “ no reasons exist for continuing to adhere to the doctrine of governmental immunity. Whatever may have been the basis for the inception of the doctrine, it is clear that no public policy considerations presently justify its retention.”

Analysis- The argument has been made that eliminating sovereign immunity will result in a massive financial burden on the state. However, this is a weak argument because insurance should cure that problem. In addition, placing the financial burden on the state would be fair, because they can afford it, rather than placing the financial burden on the individual alone.

Disposition- Reversed

Motion on Rehearing-

Issue: Whether this decision should apply (1) only to cases arising in the future; (2) to cases arising in the future and to the case at bar, or (3) to cases arising in the future, to the case at bar and to all similar pending actions.

Holding- Should only apply to cases arising in the future so that the government may have ample notice and obtain liability insurance.

40

Page 41: Torts 2 Case Briefs

Notes- - Immunities

o Immunity is a defense that bars the P’s claim even if the D would otherwise be liable and is based on the status of the D or the legal relationship between the P and D.

o Immunity may take the form of an affirmative defense or as a preclusion of subject matter jurisdiction (governmental).

o Immunity is in effect the freedoms liability for conduct that otherwise create liability. o Types:

Sovereign immunity/ governmental immunity Charitable immunity- old rule- charitable institutions had immunity under common law because

without this immunity, these institutions wouldn’t offer their services for the public good. Has largely been abolished or severely restricted (almost no longer exists).

Family immunities- an area which has been diminishing in importance. Ex- husbands and wives could not sue each other in common law, because the husband and wife had a single identity. Because the wife’s identity was subsumed into the husband’s, it wouldn’t make sense for the husband to sue himself.

Employer immunity in tort (Worker’s Comp.) - Hicks v. State

o Should New Mexico retain the doctrine of sovereign immunity in tort claims against the state? NO!!! Not any more

o Policies behind sovereign immunity Govt can do its job properly as opposed to being distracted Public inconvenience Keeps taxes lower Cost of insurance is too much

o Policies for abolition Individual shouldn’t bear the burden alone Govt can get insurance Matter of fairness, equity, and justice Public should bear the loss of governmental mistakes as opposed to the injured individual bear

the entire cost/injury alone Common law rule created by the court, that the court has the power to abolish if it so chooses

(if it is a common law rule, then it seems as though, the judges have the authority to change it. The other philosophy is that major changes in the law should come from legislatures that are elected to change it by the democracy.)

Spreading the costs of the injury so that no one individual is catastrophically affected through no fault of his own.

o Proprietary v. governmental functions of municipal (city, county and other local government units) Proprietary function- the govt is reliable.

o Note 1 Most states have at least partially abolished sovereign immunity

o Note 2o Note 3 o Note 4 – federalism and the federal constitution

Can’t sue a state in federal court in tort Generally a matter of state law and fed courts will not take jurisdiction except in the case of civil

rights cases

41

Page 42: Torts 2 Case Briefs

- Federal Government Immunity governed by the Federal Tort Claims Act (FTCA)o FTCA-

Involves claims against the federal govt, agencies or employeeso Jurisdiction

Exclusively in the federal district courts for money damages by torts of federal employees acting in the scope of employment

o Time Limits 2 years to file a claim with the agency (maximum time) Must sue within 6 months of final denial of claim (file suit in federal court) If agency fails to act on timely claim, claimant may file suit after 6 months (but claim must be

“timely”) o No jury trial against the U.S. o If U.S. is liable , it is liable for the same damages that would be available against a private person, except:

No punitive damages No pre-judgment interest

o In wrongful death claims , if only damages under applicable state law are punitive, U.S. is liable for compensatory damages instead (measured by the pecuniary damages)

Compensatory instead of punitive o Exceptions:

No liability to the US for torts that: Are based on employees exercising due care in executing statutes or regulations

(without regard to validity of the statute of regulation) Or arise from performance or not of discretionary functions

No claims for assault, battery, false imprisonment, malicious prosecution, abuse of process, defamation, misrepresentation, deceit or interference with contracts

o Exceptions to the exceptions : Assault, battery, false imprisonment, false arrest, abuse of process, or malicious prosecution

with regard to actions or omissions by federal law enforcement or investigative officers (with law enforcement powers)

o No claims arising from combat activities during war timeo No claims arising in foreign countrieso No claims for Treasury fiscal operations or monetary regulation

42

Page 43: Torts 2 Case Briefs

2. FEDERAL GOVERNMENT

a. FEDERAL TORT CLAIMS ACT

UNITED STATES v. GAUBERT SCOTUS, 1991

Page 478

Procedural History- Gaubert filed an action in the DC against the US under the Federal Tort Claims Act (FTCA), seeking $100 million damages for the lost value of his shares and for the property forfeited under his personal guarantee on the ground that the FHLBB and Dallas division were negligent in carrying out their supervisory activities. The DC granted D’s MTD for lack of SMJ on the ground that the regulator’s actions fell within the discretionary function exception to the FTCA. Relying on Indian Towing Co. v. U.S., the Court of Appeals found that the claims concerning the regulators’ activities after they assumed a supervisory role in IASA’s day-to-day affairs were not “policy decisions,” which fall within the exception, but were “operational actions,” which it believed were not within the exception.

Facts- a federal statute authorized the Federal Home Loan Bank Board (FHLBB) to proscribe rules and regulations governing federal savings and loan associations. During the savings and loan crisis of the FHLBB and the branch in Dallas oversaw certain operations of Independent American Savings Association (IASA), but instituted no formal action against the institutions. At the FHLBB’s request, respondent Gaubert removed himself from IASA’s management and posted $25 million as security for his personal guarantee that IASA’s net worth would exceed regulatory minimums. Later the regulators threatened to close IASA unless all management resigned. They did so and were then replaced. Thereafter, FHLBB-Dallas became more involved in IASA’s everyday business activity. Although IASA was thought to be financially stable while Guabert managed it, after a few months the new directors announced that IASA has a substantial negative net worth, and the Federal Savings and Loan Insurance Corporation assumed receivership of the institution.

Issue(s)- Did the Court of Appeals err when it reversed in part on the ground that the claims concerning the regulators’ activities after they assumed a supervisory role in IASA’s day-to-day affairs were not “policy decisions,” which fall within the discretionary function exception, but were “operational actions,” which it believed were not within the exception?

Holding- Yes. A discretionary act is one that involves choice or judgment; there is nothing in that description that refers exclusively to policymaking or planning functions.

Rule(s)- The exception covers only acts that are discretionary in nature, acts that “involve an element of judgment or choice,” and “it is the nature of the conduct, rather than the status of the actor” that governs whether the exception applies.

-The requirement of judgment or choice is not satisfied if a “federal statute, regulation, or policy specifically prescribes a course of action for the employee to follow,” because “the employee has no rightful option but to adhere to the directive.”-Discretionary function exception is only intended to “prevent judicial second-guessing of legislative and administrative decisions grounded in social, economic, and political policy through the medium of an action in tort.”

The exception only “protects governmental actions and decisions based on considerations of public policy.”-When established governmental policy, as expressed or implied by statute, regulation, or agency guidelines, allows a Government agent to exercise discretion, it must be presumed that the agent’s acts are grounded in policy when exercising that discretion.

Analysis-

43

Page 44: Torts 2 Case Briefs

Disposition- Reversed and remanded

Case Notes- - Note 3 - Note 2 - Note 1- Note 4-5

b. JUDICIALLY CREATED IMMUNITY

UNITED STATES v. JOHNSON SCOTUS, 1987

Page 486

Procedural History- Respondent, Johnson’s wife, sought damages from the US on the ground that the FAA flight controllers negligently caused her husband’s death. The DC dismissed the complaint relying on the Feres decision. The Court of Appeals for the 11th Circuit reversed. The court found that the evolution of the doctrine since the Feres decision warranted a qualification of the original holding according to the status of the alleged tortfeasor. The court found “absolutely no hint that the conduct of any alleged tortfeasor even remotely connected to the military will be scrutinized if this case proceeds to trial.” Accordingly it found that Feres did not bar respondent’s suit.

Facts- Johnson was a pilot for the US Coast Guard. Johnson’s Coast Guard Station received a distress call from a boat lost in the area. Johnson and a crew of several other Coast Guard members were dispatched to search for the vessel. Inclement weather decreased the visibility, so Johnson requested radar assistance from the Federal Aviation Administration (FAA), a civilian agency of the Federal Govt. the FAA controllers assumed positive radar control over the helicopter. Shortly thereafter, the helicopter crashed into the side of a mountain. Johnson, along with all of the other crew members, was killed in the crash.

Issue(s)- Did the Court of Appeals err in concluding that the Feres Doctrine did not bar recovery when negligence is alleged on the part of a Federal Government Employee who is not a member of the military?

Holding- Yes. There is no doubt that Johnson was killed while performing a rescue mission on the high seas, his injuries arose directly from that mission; his wife continues to receive death benefits; because Johnson was acting pursuant to the standard of operating procedures of the Coast Guard, the potential that this suit could implicate military discipline is substantial. The Feres Doctrine should have been applied.

Rule(s)- Feres Fatual Paradigm- Exists when a service member alleges negligence on the part of another member of the military. When this is present, the question becomes, “Did the injury arise out of or during the course of an activity incident to service?” The government is not liable under the FTCA for injuries of servicemen where the injuries arise out of or are in the course of activity incident to service.3 broad rationales underlying the Feres decision:1. The relationship between the Government and members of its armed forces is “distinctively federal in character”; 2. Generous statutory disability and death benefits are provided as a remedy;3. These are the types of claims that, if generally permitted, would involve the judiciary in sensitive military affairs at the expense of military discipline and effectiveness.

Analysis- Dissent says- READ THE STATUTE AS IT WAS WRITTEN!! This would render the federal government liable to any individual, servicemen included, for the negligence of a government employee.

44

Page 45: Torts 2 Case Briefs

Disposition- Reversed and remanded

Notes- - Issue: Does a military employee of the U.S. have FTCA claim against the government for negligent harm caused

by a civilian federal employee, when the injury occurs while the military employee is involved in a service activity?

- 11th Circuit held in this case: Feres doctrine applies only to bar claims of military personnel when injured by other military personal, not when injured by civilian employees of the federal government.

- Feres doctrine, according to the S.Ct., in this case: bars tort liability for the govt in any case in which service member is injured arising out od or in the course of activity incident to service, whether the injury is casued by civilian or military government

- Rationale: o Military employment is by its nature risky. Site of injury should not affect govt’s liabilityo Service related injuries are subject to statutory benefits, so a remedy is available

- Congress has not amended the statute providing for benefits to injured service personnel since the Feres doctrine was declared.

- Congress was unlikely to have intended both tort recovery and recovery under the Veteran’s Benefit Act - To permit torts liability in such cases would involve the judiciary in matters best left to military. Judicial

involvement could negatively affect military discipline and effectiveness. - Scalia: The statutory language is plain. Congress did not create an exception governing military personnel except

in combat situations. o No presumption as to Congress’ intent in passing in the FTCA should be drawn from the lack of

amendment by subsequent Congressesso None of the supporting rationales offered by the majority withstand scrutiny

3. IMMUNITY OF GOVERNMENTAL OFFICERS AND EMPLOYEES- Federal Liability Reform and Tort Compensation Act of 1988. - Absent violation of Constitution or federal stature, federal employees are immune upon the employee’s request,

the attorney general certifies that the employee was acting within the scope of employ- State Immunity

o Absolute immunity generally provided to judges, prosecutors, and high-ranking executives, acting within powers of their offices.

o Qualified immunity is usually provided to other employees for torts committed within the scope of their employment if they act in good faith belief that the acts are lawful

o Generally state or local governments cover all or part of judgments against employees acting within scope.

4. THE “PUBLIC DUTY” DOCTRINE

RISS v. CITY OF NEW YORKNew York Court of Appeals, 1968

Page 495

Procedural History- Trial court dismissed the complaint. Appellate division affirmed.

Facts- Linda Riss was stalked for over 6 months by a psycho, Burton Pugach. He repeatedly threatened to kill or injure her if she did not go out with him. In fear for her life, she called the police, but they did nothing. Linda subsequently was

45

Page 46: Torts 2 Case Briefs

engaged to another man. At their engagement party, the psycho called her and told her it was her “last chance.” She then called the police again and was refused assistance. The next day Pugach hired a “thug” to throw lye in Linda’s face. She was blind in one eye, lost substantial vision in the other, and her face was scarred. After the assault, she was afforded round-the-clock protection for the next 3.5 years from the police.

Issue(s)- Did the appellate division err in affirming the trial court’s decision to dismiss the complaint because the police department owed the public no duty to prevent this kind of injury?

Holding- No. It would be impractical for the Court to carve out an area of tort liability for police protection to members of the public. Police resources are limited by the resources of the community and by a legislative-executive decision as to how those resources may be deployed.

Rule(s)- Police officers can’t be held liable to members of the public who are injured by criminal third parties. Before such extension of responsibilities should be dictated by the indirect imposition of tort liabilities, there should be a legislative determination that that should be the scope of public responsibility.

-However, this immunity would not apply if the police authorities undertook responsibilities to particular members of the public and then exposed them, without adequate protection, to the risks which then materialize into actual loss.

Analysis-

Disposition- Affirmed

Notes- - Public duty doctrine

o No duty to rescueo No general duty to protect third parties from harm by third personso No duties to protect specific individuals from harm where there is a duty owed to the general publico Some state limit the public duty doctrine to police protection

Riss:Local govt does not owe a duty to one individual to protect them from harm caused by criminal wrongdoers.

Court distinguished this kind of govt activity from others activities such as duties that arise in the course of operating public

The court did not feel that it was appropriate to become involved with the choices of eth allocation of resources by local govt. The level of police protection was better left to leg decisions. Court is not competent to make such determinations.

However, if the police authorities undertake responsibilities to specific members of the public, the govt can be held for negligence in failing to act reasonably performing such undertakings.

46

Page 47: Torts 2 Case Briefs

B. FAMILY IMMUNITIES

PRICE v. PRICESupreme Court of Texas, 1987.

732 S.W.2d 316

Procedural History- Summary Judgment was granted for defendant, Duane Price, in this negligence action. Court of Appeals affirmed the judgment. Kimberly Price appeals.

Facts- Kimberly Price was injured in July of 1983 when the motorcycle of which she was a passenger, driven by her future husband, Duane Price, collided with a truck. Kimberly married Duane 6 months later and brought suit against Duane and the other truck driver, alleging that their negligence combined to cause her injury. The driver of the truck and his employer settled out of court, but the court issued summary judgment for Duane, relying on the doctrine that one spouse could not sue another for negligence (Interspousal Immunity).

Issue(s)- Did the Court of Appeals err in ruling that the doctrine of interspousal immunity barred Kimberly Price from recovering against her husband, Duane Price?

Holding- Yes. The interspousal immunity doctrine is no longer good law and his abolished. The Married Women Acts gave wives the right to own, acquire and dispose of property, to contract, and to have the right to sue in regards to their own property in contracts. The Common law holding that a “husband and wife are one person” is no longer valid. It is difficult to fathom how denying a forum for the redress of any wrong could be said to encourage domestic tranquility. The Doctrine of Spousal Immunity is abolished completely as to any cause of action.

Rule(s)- The common law rule for interspousal immunity is no longer good law. The Doctrine of Spousal Immunity is abolished completely as to any cause of action.

Traditional justifications for the rule (of spousal immunity): legal unity, husband’s status, spousal harmony, and collusive suits

Disposition- Reversed and Remanded.

BROADBENT v. BROADBENTSupreme Court of Arizona, 1995.

Page 503

Procedural History- Trial Court granted the mother, Laura Broadbent’s, Motion for Summary Judgment on the grounds that the parental immunity doctrine applied to the facts of this case.

Facts- Christopher Broadbent, Plaintiff/Appellant, and his mother, Laura Broadbent, Defendant/Appallee, went swimming at their residence on April 13, 1984. Christopher was wearing his floaties, as he was only 2 and ½ years old and did not know how to swim. Laura left Christopher by the side of the pool to go answer the telephone even though she had seen him remove his floaties. Laura talked on the phone for 5-10 minutes before she noticed that she could not see Christopher from where she was talking. She found him floating in the deep end of the pool and began CPR on him. The paramedics arrived shortly thereafter and Christopher was finally revived at the hospital. Due to the long length of oxygen depravation, Christopher suffered sever brain damage and now has lost his motor skills and voluntary movement.

Issue(s)- Did the Trial Court err in granting summary judgment for the Defendant under the parental immunity doctrine?

47

Page 48: Torts 2 Case Briefs

Holding- Yes. The trier of fact must apply the “Reasonable Parent” Standard. The trier of fact must decide whether the mother in leaving the 2 and ½ year old child unattended next to the swimming pool is reasonable or prudent.

Rule(s)- Exceptions to Parental Immunity: if the parent is acting outside his parental role and within the scope of employment; if the parent acts willfully, wantonly, or recklessly; if the child is emancipated; if the child or parent dies; if a third party is liable for the tort, then the immunity of the parent does not protect that third party; and if the tortfeasor is standing in loco parentis, such as a grandparent, foster parent, or teacher, the immunity does not apply.Reasons in Support of Parental Immunity:

(1) Suing one’s parents would disturb domestic tranquility;(2) Suing one’s parents would create a danger of fraud and collusion;(3) Awarding damages to the child would deplete family resources;(4) Awarding damages to the child would benefit the parent if the child predeceases the parent and the

parent inherits the child’s damages;(5) Suing one’s parents would interfere with parental care, discipline, and control.

“Reasonable Parent” Standard for Parent-Child Suits- A parent’s conduct is judged by whether that parent’s conduct comported with that of a reasonable and prudent parent in a similar situation!!! A parent is not immune from liability for tortious conduct directed toward his child solely by reason of that relationship. And, a parent is not liable for an act or omission that injured his child if the parent acted as a reasonable and prudent parent in the situation would.-The “Reasonable Parent” standard attempts to fashion an objective standard that does not result in second-guessing parent in the management of their family affairs.

-Overrules Sandoval, which created the “duty to the world at large versus duty to the child alone” distinction. Also overrules the Goller Standard and Test.

Reasoning/Analysis-

Disposition- Reversed and Remanded

Parental Immunity Notes

Under what circumstances should a parent be held liable in tort to a child for injury caused by parent’s fault? American common law rule – parents are immune from tort suits by minor or emancipated children.

Policy reasons: family harmony, parental discipline, family resources for other family members (spec. other children), and undue intrusion by the states into matters of the family. Parental inheritance by wrongdoing parent. (so parent gets to keep the money in the 1st place, assuming the parent wouldn’t spend the money on something else. (see above)

Rule eroded over time – no immunity for intentional torts, no immunity for reckless, willful torts, no immunity in auto accidents, then no immunity when duty is one owed generally to others. This continued to the point that parents are liable exept with reference to matters of legal obligations. Continued to the point of where the law was at the time of Broadbent

Broadbent case:Parental immunity is abolished. Parents are liable in tort when they have failed to act as reasonable parents in the situation. -Insurance is gen available to cover liability or suit is not brought. Parents are gen the parties who actually initiate the suit against themselves. (does create a conflict of interests) -Likelihood of inheritance by parent is remote. Reasonable parent rule permits sufficient discretion and flexibility to accommodate different styles of parenting.

48

Page 49: Torts 2 Case Briefs

-serious injuries are mor likely to occur in exercising parental authority when the parent acts in a willful pr wanton manner (such as severely beating a child) and such conduct should not be protected.-

CHAPTER 13 - MEDICAL MALPRACTICE

PROFESSIONAL STANDARD OF CARE

MELVILLE v. SOUTHWARDSupreme Court of CO, 1990

Page 512

Procedural History- Trial Court awarded P a $56,000 award for malpractice of a podiatrist, which resulted in a permanent foot injury. Court of Appeals overruled. Plaintiff appeals.

Facts- In 1980, P consulted with D about an ingrown toenail. D removed the ingrown toenail and also suggested that P have a metatarsal osteotomy to relieve discomfort. The podiatrist performed the surgery a month later in his office. One week later, the P returned to D’s office for a checkup. The doctor said that he did not like the looks of the foot, and provided P an additional antibiotic. After another week, P complained that her foot was red, swollen, and painful. D told her to increase the amount of vinegar and soak the foot more frequently. Two days later, D told P that her wound was healing properly. The next day P noticed fluid coming out of the area near the incision and called her family physician, who in turn referred her to Dr. Barnard, an ortho surgeon. Barnard testified that the osteotomy procedure was below the standard of care for 2 reasons: (1) Because it was unnecessary and (2) the procedure was performed in an unsterile office environment. He also testified that the post-operative standard of care was also below the standard.

Issue(s)- Did the trial court err in allowing an orthopedic surgeon to be an expert witness as to the degree of care owed by another specialty, a podiatrist? Was the Court of Appeals correct in ruling that the testimony offered by Dr. Bernard with regard to the standard of care during and after the operation was nothing more than opinion testimony?

Holding- Yes. The P failed to establish an evidentiary foundation that Dr. Bernard, by reason of his knowledge, skill, experience, training, or education, was so substantially familiar with the standard of care for podiatric surgery as to render his opinion testimony as well informed as that of a podiatrist. Nor did the P establish that the standard of care for the procedure performed was identical for both orthopedics and podiatrists.

Rule(s)- In a medical malpractice case, the P must establish that the D failed to conform to the standard of care ordinarily possessed and exercised by members of the same school of medicine practiced by the Defendant. -The standard of care in a medical malpractice case is measured by whether a reasonably careful physician of the same school of medicine as the D would have acted in the same manner as did the D in treating and caring for the P. -In order to establish what the controlling standard of care owed is, the P must introduce expert testimony. The court must make 2 preliminary determinations regarding the expert testimony: (1) Will the expert testimony assist the trier of fact in understanding the evidence or in determining a fact in issue; (2) Is the expert witness properly qualified by knowledge, skill, experience, training, or education to offer an opinion on the issue in question.

Reasoning/Analysis-

Disposition- Remanded to the district court for a new trial.

49

Page 50: Torts 2 Case Briefs

Notes- Negligence: (must show all of these)

o Duty- Standard of Care (Other physicians in the same school of medicine – whether a reasonably careful physician (mentally competent person) of the same school of medicine would have acted the same way)

o Breach- o Cause in fact- o Legal Cause- o Damages

In medical malpractice cases the standard of care is measured by the skills ordinarily possessed and exercised by members of the same field or school of medicine as practiced by the defendant. (think about this as what the bas level of minimum competency accepted within the profession, not what the skill level of the “average” m.d. would be)

Duty is to use reasonable care, but reasonable care is defined by reference to the professional standard of care for the applicable profession, specialty or subspecialty of the profession involved. (where one has a “recognized profession”- licensed or regulated- have established standards of care, or rules of ethics)

Other courts: o The standard of care is the standard of a minimally competent practitioner in that specialty or field.

(What is reasonable in that specialty?) The test is then, what would a minimally competent or qualified doctor in that field have done under the circumstances?

Note: Not the “average doctor.” (1/2 would be below that standard)o In other words,

Expert testimony:o Ordinarily needed to establish what the standard of care iso The expert must be familiar with the defendant’s specialty (usually in the same specialty) or be in a

specialty which has substantially identical standards of care. o Note: the expert’s personal opinion of what should have been done is not sufficient. The expert must

establish what the standard of care is for the specialty, not his personal standard or preference. There are two parts to the expert opinion needed:

o What is the Standard of care applicable to the D?o Did the D fail to meet the applicable SOC?

P must establish both- and then must also establish cause in fact and legal cause As to cause in fact, that often will require expert testimony as well, but is really a separate matter. The expert need not be in the same specialty to establish cause in fact Professional standards in other professions:

o lawyers, accountants, architects, engineers National v. Local standard

o Specialties- generally nationalo General practices-

Most states- either “modified” local (i.e., same or similar communities) or national But generally can take in account availability of facilities- rural vs. city

Alternative medicine:- Note 5 page 519 o Holistic, eastern, etc.

HARNISH v. CHILDREN’S HOSPITAL MEDICAL CENTERSupreme Judicial Court of MA, 1982.

Page 520

50

Page 51: Torts 2 Case Briefs

Procedural History- A medical malpractice tribunal concluded that the P’s offer of proof was insufficient to raise a question appropriate for judicial inquiry. The action was dismissed after P failed to produce a bond. P appeals the dismissal.

Facts- P underwent an operation to remove a tumor in her neck. During the procedure, her hypoglossal nerve was severed, allegedly resulting in permanent and almost total loss of her tongue. P’s complaint charges the physicians and the hospital with failing to inform her before surgery of the risk of loss of tongue function. Further, the complaint alleges that the loss of the tongue function was a material and foreseeable risk, and that, had she been informed of the risk, she would not have consented to the operation. A medical malpractice tribunal concluded that the P’s offer of proof was insufficient to raise a question appropriate for judicial inquiry.

Issue(s)- Did the medical malpractice tribunal err in ruling that P’s offer of proof was insufficient to raise a question appropriate for judicial inquiry?

Holding- Yes. A physician’s failure to divulge in a reasonable manner to a competent adult patient sufficient information to enable the patient to make an informed judgment on whether to give or withhold consent to a medical or surgical procedure constitutes professional misconduct. P must only prove that had the proper information been provided neither she nor a reasonable person in similar circumstances would have undergone the procedure.

Rule(s)-The Rule of Liability- The performance of a surgical procedure by a physician without the patient’s consent constitutes professional misconduct.

Doctrine of Informed Consent- “It is the prerogative of the patient, not the physician, to determine the direction in which his interests lie.”

-Every competent adult has the right to forgo treatment, or even cure, if it entails what for him are intolerable consequences or risks however unwise his sense of values may be in the eyes of the medical profession.

A physician owes to his patient the duty to disclose in a reasonable manner all significant medical information that the physician possesses or reasonably should possess that is material to an intelligent decision by the patient whether to undergo a proposed procedure.

The information that a physician reasonably should possess is that information possessed by the average qualified physician or, in the case of a specialty, by the average qualified physician practicing that specialty.

The extent to which he must share that information with his patient depends upon what information he should reasonably recognize is material to the plaintiff’s decision. Materiality may be said to be the significance of a reasonable person, in what the physicians knows or should know is his patient’s position, would attach to the disclosed risk or risks in deciding whether to submit or not submit to the treatment.

Reasoning/Analysis-

Disposition- Reverse dismissal of claim against Dr. Holmes and Dr. Mulliken, uphold the dismissal against the hospital and Dr. Gilman.

Notes- - Physician should disclose the material risks that may occur during the proposed procedure - Material risks will vary depending on the type of surgery, whether it be elective or life saving. - Doctor must disclose the risks that are material and inherent in the procedure, in addition to the risks of non

treatment and alternative treatments. - Informed Consent

o Battery or Negligence

51

Page 52: Torts 2 Case Briefs

Battery if no consent at all is given and the touching is intentional and harmful Negligence if consent to medical treatment by a physician, but the consent is based on

inadequate disclosure of risks.o Why have informed consent?

Protects the patient’s sense of self-autonomy or bodily integrity – the patient should have the right to say what should be done to him. (not to individuals specifically, it is more general. Reasonable person standard)

Protects patient’s decision making - Harnish v. Children’s Medical Center

o Physician is guilty of malpractice when the physician performs surgery without the patient’s consent o P must prove the failure of the Dr. to disclose the material facts. In addition, she must prove that but for

the Dr. telling her, she wouldn’t have gone through with the surgery. o Then P must prove that a reasonable person would have acted the same given the circumstances

- Policy: Patient’s interest o Physician is under a duty to disclose in a reasonable manner to competent adult patients sufficient

information to enable patient to make an informed judgment as to whether to undergo medical or surgical treatment. (promotes more patient centered autonomy as opposed to the other side)- half the states- patient autonomy standard

- Other side- what would a reasonable physician disclose to a patient as being a material risk - Disclosure required is: all significant medical information possessed by physician or that he or she reasonable

possesses, that is material to an intelligent decision by the patient whether to undergo a proposed. – Other half the states- physician centered standard- more modern trend rule.

- What the physician should known in this context must be established by expert testimony (what knowledge should be possessed in the doctor’s field of practice)

- Materiality determination- what information has significance in the decision making process to one in the patient’s situation as known to the physician or what as it should be known to the physician

- Materiality determination DOES NOT require expert testimony - Factors to be disclosed: (merely an illustrative list- not exclusive or exhaustive)

o Nature of conditiono Nature and probability of riskso Benefits expectedo Uncertainty of resultso Whether, if relevant, results are reversibleo Alternative procedures with the attendant risks and benefits

- Court adopts “patient-centered” approach- Harnish Caseo Reasonableness of disclosure is based on what a “reasonable patient” would need to know

- About half the state have “physician-centered approach”o What would a reasonable physician disclose? (or what is customarily disclosed)

Expert testimony to establish what is customary in the field- Which approach is better?

o Patient center approach is more uncertain o We are trying to balance the two-

- Duty, breach, cause in fact (But for test…), Legal cause (was it foreseeable?) - If the physician then breaches the duty of informed consent, patient still must prove

o Cause in fact of harm as result of undergoing the procedure and the occurrence of that harm from an undisclosed risk that should have been disclosed.

- P must also prove that a reasonable person would not have undergone the procedure- Problem: is there a range of acceptable reasonable responses to risks?

o In other words, do all reasonable people have the same degree of risk tolerance?

52

Page 53: Torts 2 Case Briefs

- Despite the rules’ obvious shortcomings, courts have generally applied an objective test regarding what must be disclosed (probably correctly so) and whether the patient would have undergone the procedure with disclosure.

CHAPTER 14 – COMMON LAW STRICT LIABILITY

ABNORMALLY DANGEROUS ACTIVITIES

RYLANDS v. FLETCHERHouse of Lords, 1868.

Page 526

Procedural History- Trial Court, Court of Exchequer, held that P had established no cause of action. The Appellate Court, the Court of Exchequer Chamber, overruled and held P was entitled to damages for the cause of action. D appeals

Facts- Plaintiff, Fletcher, mined a piece of property that he occupies. The Defendant, Rylands, et al., are the owners of a mill in P’s neighborhood. The defendants hired an engineer and contractor to build a reservoir to hold water under their land. When they began to fill the reservoir with water, the water breached the vertical shafts and spewed onto P’s mines, causing considerable damage.

Issue(s)- Did the Court of Appeal err in ruling that the P had presented a cause of action which entitled him to damages?

Holding- No. A person who, for his own purposes, brings on his land and collect and keeps there anything likely to do mischief if it escapes, must keep it in at his peril; and if he does not do so, is prima facie answerable for all the damage which the natural consequence of its escape.

Rule(s)- A person who, for his own purposes, brings on his land and collect and keeps there anything likely to do mischief if it escapes, must keep it in at his peril; and if he does not do so, is prima facie answerable for all the damage which the natural consequence of its escape.

Reasoning/Analysis-

Disposition- Affirmed

Strict Liability Notes- Liability without fault- Animals, Abnormally Dangerous Activities, Products Liability

- Animals:o Under English Common law- liability without fault for damages caused by trespassing animals (livestock)

- American common law- o fencing out- rancher oriented- if you want to hold your neighbors liable, then you have to have a

fence around your propertyo fencing in- farmer oriented – if you want to hold your neighbors liable, then you must fence your

animals in

53

Page 54: Torts 2 Case Briefs

o local variations in same state o Wild animals- Both English and American common law impose strict liabilityo Domestic Animals-

Strict liability if reason to know of vicious propensity Otherwise must prove negligence

o When animal owner is subject to strict liability, legal cause must still be established, i.e. the harm must be of the type reasonably foreseeable given the nature of the animal and plaintiff must be reasonably foreseeable

o In modern applications of the strict liability rule, comparative negligence may be used as a defense - Abnormally Dangerous Activities

o One who brings onto his land something which poses risk of harm to others, should the thing escape from the owner’s land, is liable without fault. – Rylands v. Fletcher

o If the engineers were negligent in the Rylands case, what arguments would the P make against the landowner?

Non delegable duty o Rylands as later interpreted and applied:

Activities that are abnormally dangerous (abnormal use or activity) Creates danger to others Give rise to strict liability for injuries cause by such activities

o Strict liability has not been universally applied by US in abnormally dangerous activities o How to determine what is abnormally dangerous?o Factors to consider in making a determination as to whether activity is abnormally dangerous:

Restatement (2d) § 520 High degree of risk of harm Likelihood that such harm will be great Effectiveness of reasonable care in avoiding the harm Common usage (more common= less likely it is to be abnormally dangerous) Appropriateness of the activity in the place where it occurs Value to the community

o The kind of harm (and person harmed) must be foreseeable Is what happened the foreseeable result of this conduct?

If No, then there is no claim o An activity is abnormally dangerous and therefore subject to strict liability if it “creates a foreseeable

and highly significant risk of physical harm even when reasonable care is exercised” and it is “not a matter of common usage.” – Restatement (3d) – can use this standard or the one above for the test

o Always considered an abnormally dangerous activity: Dynamiting Pile driving

o Not really looking at the danger to yourself in strict liability. Looking at dangers to other people in performing a certain activity

- Purpose of strict liability ruleo Person who is not engaging in the activity should not have to bear the loss. It is more equitable for the

person who is engaging in the abnormally dangerous activity to bear the loss because they are the person that is creating the risk of loss to others.

54

Page 55: Torts 2 Case Briefs

SEIGLER v. KUHLMANSupreme Court of Washington, 1972

Page 530

Procedural History- Trial Court issued directed judgment for defendants. P appealed to Court of Appeals, which affirmed. P appealed to S.C. of Washington which reverses.

Facts- 17 year old Carol House died in a gasoline explosion after her car encountered a pool of thousands of gallons of gasoline. On the early evening of November 22, 1967, while driving home from after school job in Olympia, she was involved in an accident with a gasoline tanker that was driven by Aaron L. Kuhlman of Pacific Intermountain Express. Kuhlman testified that he was driving down hill when he felt a jerk and notice in the rearview mirrors that the trailer had disengaged from his truck. The tank trailer catapulted off the freeway through a chain-link fence and landed upside down on Capital Lake Drive. There was no evidence of impact with the vehicle, except for the fact that the left front headlight was broken.

Issue(s)- Did the trial court err in refusing to give an instruction on res ispa loquitor? Should the trial court and Appellate Court have used strict liability arising as a matter of law from the circumstances of the event?

Holding-Yes. This is a case of strict liability as the activity that caused the injury is considered abnormally dangerous. Transporting gasoline as freight carries a high degree of risk; it is a risk of great harm and injury; and it creates dangers that cannot be eliminated by the exercise of reasonable care.

Rule(s)- A person who, for his own purposes, brings on his land and collect and keeps there anything likely to do mischief if it escapes, must keep it in at his peril; and if he does not do so, is prima facie answerable for all the damage which the natural consequence of its escape.Restatement of Torts, Section 519-Strict Liability

(1) One who carries on an abnormally dangerous activity is subject to liability for harm to the person, land or chattels of another resulting from the activity, although he has exercised the utmost care to prevent such harm.

(2) Such strict liability is limited to the kind of harm, the risk of which makes the activity abnormally dangerousRestatement of Torts, Section 520- What Constitutes Abnormally Dangerous Act?In determining whether activity is abnormally dangerous, the following factors are to be considered:

(a) Whether the activity involves a high degree of risk of harm to the person, land or chattels of others;(b) Whether the gravity of the harm which may result from it is likely to be great;(c) Whether the risk cannot be eliminated by the exercise of reasonable care;(d) Whether the activity is not a matter of common usage;(e) Whether the activity is inappropriate to the place where it is carried on; and(f) The value of the activity to the community.

Reasoning/Analysis-

Disposition- Reversed

55

Page 56: Torts 2 Case Briefs

CHAPTER 15 - PRODUCTS LIABILITY

EMERGENCE OF STRICT TORT LIABILITY

GREENMAN v. YUBA POWER PRODUCTS, INC.Supreme Court of CA, 1963.

Page 540

Procedural History- Trial Court ruled that there was no evidence that the retailer was negligent or had breached any express warranty and that the manufacturer was not liable for any breach of an implied warranty. Trial Court jury returned a verdict of $65,000 for plaintiff against the manufacturer, but returned a verdict for the retailer against the P. Plaintiff appeals, seeking a reversal of the judgment in favor of the retailer.

Facts- Plaintiff brought this action for damages against the retailer and manufacturer of Shopsmith, a combination power tool that could be used as a saw, drill, and wood lathe. His wife bought P a Shopsmith as a Christmas present in 1955. In 1957, he bought the necessary attachments needed for a project he was working on. After he had worked on a piece of wood several times, the attachment flew out of the wood and struck him in the head. 10 and ½ months later he notified the retailer and manufacturer of his claim for breaches of warranties and filed a claim for said breaches and negligence.Plaintiff introduced adequate evidence that his injuries were caused by defective design and construction of the Shopsmith. His experts testified that inadequate set screws were used to hold parts of the machine together so that normal vibration caused the tailstock of the lathe to move away from the piece of wood being turned permitting it to fly out of the lathe.

Issue(s)- Did the trial court err in granting judgment for the plaintiff against the manufacturer in that Plaintiff did not give it notice of the breach of warranty within a reasonable time, therefore barring it from action under 1769 of the Civil Code?

Holding- No. Even if Plaintiff did not give timely notice of breach of warranty to the manufacturer, his cause of action on the representations contained in the brochure was not barred. The purpose of strict tort liability is to ensure that the costs of the injuries resulting from defective products are borne by the manufacturers who put such products on the market rather than the injured person who are powerless in protecting themselves. To establish the manufacturer’s liability, it was sufficient that P proved that he was injured while using the Shopsmith in a way it was intended to be used as a result of a defect in the design and manufacture, of which P was not aware, that made the Shopsmith unsafe for its intended use.

Rule(s)- A manufacturer is strictly liable in tort when an article he place on the market, knowing that it is to be used without inspection for defects, proves to have a defect that causes injury to a human being.

Reasoning/Analysis-

Disposition- Affirmed.

Products Liability Notes- A specific type of tort law- Narrow and focused on a specific range of cases- Refers to civil liability for injuries caused by defective products (humanly processed items, artificial, or involve

human activity to bring to market) 56

Page 57: Torts 2 Case Briefs

DEFECT

GRAY v. MANITOWOC CO., INC.United States Court of Appeals, 5th Circuit, 1985.

Page 544

Procedural History- Earnest Gray and his wife Hughlene Gray, Plaintiffs, brought an action against the Manitowoc Company, for various things arising from personal injuries sustained by Earnest when he was struck by the boom of a construction crane manufactured by Manitowoc. His wife joined the action alleging loss of consortium and companionship. They sued under theories of strict liability, implied warranty, and negligence by asserting that Gray’s injuries were caused by a defect in the design of the crane and Manitowoc had failed to provide adequate warning of the defect. -Trial Court jury returned a verdict in favor of the Gray’s, Plaintiffs. Defendant moved for a j.n.o.v. which was denied. Defendant, Manitowoc, appeals.

Facts- Plaintiff was struck in 2 separate incidents by the butt end of the boom of a Manitowoc 4100W crane while working as an ironworker foreman on a project near Port Gibson, MS. These incidents occurred while Gray’s crew was changing sections of the crane’s boom and had placed the boom in a roughly parallel position to the ground. Gray was standing on the left side of the crane supervising when the crane operator swung the lowered boom in Gray’s direction, striking Gray in the back. Testimony at trial established that the operator’s vision to the left side of the crane is obscured by the boom when the crane is in boom-down position. Gray contends that Manitowoc, in addition to already requiring signalcallers on the ground, should have equipped the crane with mirrors, or closed circuit television cameras.

Issue(s)- Did the trial court err in allowing the jury verdict for the plaintiffs to stand? Did the evidence establish that there was a latent hazard, as required by MS law? Should the j.n.o.v. have been granted?

Holding- Yes. The record does not support a finding that the blind spot in the 4100W was a latent hazard. The evidence was overwhelming that the existence of this blind spot was common knowledge in the construction industry. No reasonable jury could have concluded that the blind spot of the 4100W was not open and obvious, nor could any reasonable jury have concluded the 4100W was dangerous to a degree not anticipated by the ordinary consumer of that product.

Rule(s)- MS version of strict liability for hazardous products says that manufacturers are not insurers of the products they produce; the existence of a product defect must be established before recovery may be obtained for a resulting injury.Consumer Expectation Test of 402A-Defective Condition- Only applies where the product is, at the time it leaves the seller’s hands, in a condition not contemplated by the ultimate consumer, which will be unreasonably dangerous to him. “Product does not meet the reasonable expectations of the ordinary consumer as to its safety.”Unreasonably Dangerous- The article sold must be dangerous to an extent beyond that which would be contemplated by the ordinary consumer who purchase it, with the ordinary knowledge common to the community as to its characteristics.

Reasoning/Analysis-

Disposition- Reverse and render judgment.

Failure to Warn?

57

Page 58: Torts 2 Case Briefs

PHILLIPS v. KIMWOOD MACHINE CO.Supreme Court of Oregon, 1974

Page 548

Procedural History- Plaintiff was injured while feeding a fiberboard into a sanding machine during the course of his employment with Pope and Talbot. The sanding machine was purchased from the defendant, Kimwood. Plaintiff brought this action on a products liability theory, contending the sanding machine was unreasonably dangerous by virtue of defective design. At the completion of testimony, trial court judge granted D’s motion for a directed verdict. Phillips appeals.

Facts- On the day of the accident, P was engaged in feeding thick sheets of fiberboard into the sander. Somehow, a thin sheet was mixed with the lot, and inserted into the machine. The machine regurgitated the thin piece of wood back at P, hitting him in the abdomen and causing him injury. P contends that the machine’s design was defective because it did not have safety devices to protect the person feeding the machine from the regurgitation of wood. There was evidence presented that established that for a relatively low price, the machines could have been installed with a safety device to stop these types of injuries. Evidence was also presented that after the accident, that these safety “teeth” were inserted on the machines by the employer.

Issue(s)- Did the trial court err in providing directed motion for Defendant?

Holding- Yes. Although the employer decided to buy the machine without the automatic feeder from D, a failure to warn can make a product unreasonably dangerous. The D, who knew that the P’s company intended to feed the machine manually and who had constructive knowledge of the machine’s propensity to regurgitate smaller pieces of wood, and who failed to warn the employer of the problem, is liable.

Rule(s)-Misdesign v. Mismanufacture- The process of determining what is unreasonably dangerously defective is much easier in the case of mismanufacture than misdesign. A dangerously defective article would be one by which a reasonable person would not put into the stream of commerce if he had knowledge of its harmful character.

-Test for what is a dangerously defective article: Whether the seller would be negligent if he sold the article knowing of the risk involved.

Seller-oriented standard v. User-oriented Standard- These standards are not inconsistent with one another as they both turn on foreseeable risks. They are two sides of the same standard. A product is defective and unreasonably dangerous when a reasonable seller would not sell the product if he knew of the risk involved or if the risks are greater than the reasonable buyer would expect.

Reasoning/Analysis-

Disposition- Reversed and Remanded

Notes-- Phillips v. Kimwood Machine Co.

o What is the appropriate test for unreasonably dangerous defect?o Consumer expectation or Risk/utility (but in hindsight)o On the other hand they like the fact that open and obvious dangers bar recovery and that the plaintiff

has the burden of proof o Mismanufacture- the product deviates from the original blueprint/specifications (can be compared to

others that are perfectly normal) o Design defect- built to specification but the design is dangerous and poses a risk of harm to consumers

58

Page 59: Torts 2 Case Briefs

o Court tries (unconvincingly) to reconcile the two approaches. Flaw is particularly apparent in the effect of open and obvious dangers.

o Most courts have moved to a risk/utility based approach for design caseso Less explicit about manufacturing defect cases, where consumer expectation still is a significant factor in

some senseo In warnings cases, courts generally adopt a negligence based approach: (information defect)

Seller/manufacturer must warn about dangers that were known or that were reasonably knowable (constructive knowledge) at the time the product was sold

o Phillips bucks this trend by attempting to make warnings cases truly strict liability: Would a reasonable manufacturer have warned if it had known what is known at the time of

trial?o Design defect: the concept makes the product unreasonably dangerouso Manufacturing defect: the produce deviated from the concept (blueprint) in such a way as to cause the

product to be unreasonably dangerous o Warnings defect: the manufacturer failed to give adequate warning about the product’s risks, failed to

provide adequate warning about proper use, etc., so that the product is unreasonably dangerous. o In manufacturing defects:

Strict liability is generally applied (with negligence being an alternative)o In design cases: jurisdictions are more divided

Strict liability using consumer expectation Strict liability using risk/utility in hindsight Strict liability using negligence:

o Reasonable alternative design available at the time of sale Reasonable design involves a risk/utility analysis, and involves cost issues, performance

issues, foreseeability of the risk o Alternative design test for design defect- strict liability- do not look at foreseeability, look at cost and

performance Cost issues- Performance issues- utility issues- is the new product as efficient as the older design? Foreseeability issues- negligence - difference between Other risks involved- are the new dangers created by creating an alternative design more

dangerous that the only risk? Value of the products use

Rx drugs- beneficial to the point that they are worth some degree of risk Aesthetics Technology- is it something that is available?

o Consumer expectation test- is this product more dangerous than a reasonable consumer would expect it to be?

o Most courts have moved to a risk/utlity based approach for design caseso Less explicit about manufacturing defect cases, where consumer expectation still is a significant factor in

some sense. o Most courts will use the negligence test

Drug Manufacturers- Duty to Warn of defects?

FELDMAN v. LEDERLE LABORATORIESSupreme Court of New Jersey, 1984.

Page 554

59

Page 60: Torts 2 Case Briefs

Procedural History- Trial Court ruled for D. Appellate Division Affirmed. SC of NJ remanded the cases to the Court of Appeals and it was again affirmed. P appeals yet again on the issue of strict liability.

Facts- Plaintiff has gray teeth as a result of taking a tetracycline drug, Declomycin. Plaintiffs father, a physician, administered her the drug when she was an infant to control upper respiratory and other types of infections. The drug was manufactured by Defendant. Plaintiff brought suit not because the drug was defective in controlling infections; rather, the suit was brought for the failure to warn about the drug’s side effect, tooth discoloration. P took the meds from 1960-63. Evidence was sharply contradicted as to whether the D knew of the side effect in 1960, but by 1962, D did know of the risk and amended its labeling. D claimed that it was not liable because it did not know of the risk at the time its literature and product were disseminated.

Issue(s)- Did the trial court err in ruling that the Doctrine of Strict Products Liability applies drug manufacturers? Was the medicine prescribed unavoidably unsafe?

Holding- No. Drug manufacturers have a duty to warn of dangers of which they know or should have known on the basis of reasonably obtainable or available knowledge. The medicine was not unsafe or unavoidably so. Drugs, like any other product, may contain defects that could have been avoided by better manufacturing or design. Whether a drug is unavoidable unsafe must be determined on a case-by-case basis. Prescription drug manufacturers should not get a “blanket immunity” from strict liability manufacturing and design defect claims under comment K.

Rule(s)- Drug manufacturers have a duty to warn of dangers of which they know or should have known on the basis of reasonably obtainable or available knowledge.-The difference between strict liability and negligence is commonly expressed by stating that in a strict liability analysis, the defendant is assumed to know of the dangerous propensity of the product; whereas in a negligence case, the P must prove that the D knew or should have known of the danger.-The question in strict liability design—defect and warning cases is whether, assuming the manufacturer knew of the defect in the product, he acted in a reasonably prudent manner in marketing the product or in providing the warnings given.-As to warnings, generally conduct should be measured by knowledge at the time the manufacturer distributed the product. Did the D know, or should he have known, of the danger, given the scientific, technological, and other information available when the product was distributed; or in other words, did he have actual or constructive knowledge?-In strict liability warning cases, the D should properly bear the burden of proving that the information was not reasonably available or obtainable and that it therefore lacked actual or constructive knowledge of the defect. Reasoning/Analysis-

Disposition- Reverse and Remanded….again

60

Page 61: Torts 2 Case Briefs

Design Defect Claims

POTTER v. CHICAGO PNEUMATIC TOOL CO.Connecticut Supreme Court, 1997.

Page 561Procedural History-

Facts- Shipyard workers claim they were injured as a result of using defectively designed pneumatic hand tools manufactured by Ds.

Issue(s)- Did the trial court err in applying the consumer expectation test to determine whether liability should be enforced? Should, as defendants urge, the court abandon that standard and require the P to prove the existence of a reasonable alternative design in order to prevail on a design defect claim?

Holding- No. The feasible alternative design requirement imposes an undue burden on Ps that might preclude otherwise valid claims from jury consideration. This would also require the P to obtain an expert witness, and in CN, it has commonly been held that an expert is not needed if the jury can infer a defect from the evidence. The availability of a feasible alternative design is a factor the P may, rather than must, prove in order to establish that a product’s risks outweigh its utlity.

Rule(s)-Consumer Expectation Test- Section 402A- A manufacturer is strictly liable for any condition not contemplated by the ultimate consumer that will be unreasonably dangerous to the consumer. Comment i of 402A- Unreasonably Dangerous- article sold must be dangerous to an extent beyond that which contemplated by the ordinary consumer who purchases it, with the ordinary knowledge common to the community as to its characteristics.OrModified Consumer Expectation Test-Balancing Test that inquires whether a product’s risks outweigh its benefits- Under the risk/utility test, the manufacturer bears the burden of proving that the product’s utility is not outweighed by its risks in light of various factors such as:1. usefulness of the product2. the likelihood and severity of the danger posed by the design3. the feasibility of an alternative design4. the financial cost of an improved design5. the ability to reduce the product’s danger without impairing its usefulness or cost6. the feasibility of spreading the loss by increasing the product’s price

-Other jurisdictions apply only a risk/utility test to determine whether a manufacturer is liable.

Reasoning/Analysis-

Disposition-Affirmed.

Notes- - If you can prove consumer expectation or reasonable alternative design theory, use either or. - HOWEVER, in MS- you have to prove BOTH consumer expectation AND reasonable alternative design theory. - State of the art- what is technologically feasible and cost effective… or… standard custom of the injury….or…

Causation- Must be proven in products liability- Cause-in-fact- remains a required element of proof, even in strict liability - Difficyl cases for plaintiffs:

61

Page 62: Torts 2 Case Briefs

o Warnings Rebuttable presumption

o Crashworthy cases Do car manufacturer’s have the duty to take into account crashes? Even from driver’s

negligence? Courts typically say yes

o Drug cases- who manufactured? - Test

o Duty Negligence? Strict liability?

o Breach Collapses with causation in a strict liability case Defect existed when the product left the manufacturer’s control and there was a causal

relationship between the injury and the defect If product was changed after it left the manufacturer’s control, then it is not the same product,

therefore no liability o Cause in fact

Injury was caused by an unreasonably dangerous product o Legal causeo Damages

- Legal cause is very rarely and issueo Foreseeability is hindsighto Would this injury have been foreseeable if the defect was known before?o Plaintiff must be foreseeable but, if hindsight test, danger need not have been foreseeable (at lease if

the use was foreseeable). However, even then, court is likely to use foreseeability in some way, as in “now that we know the risk, is the risk one that would be considered reasonably foreseeable in the future.”

Potter v. Chicago Pneumatic Tool Co- What is the appropriate test for defect?

o Consumer expectationso Risk Utilityo Combination?

- Restatement 3d (general approach in the US, but not the strongest- there is a lot of mixture)o Unreasonably dangerous when the foreseeable risk of harm could have been avoided or reduced by

adoption of a reasonable alternative design - Either or approach – Restatement 2d

o Consumer expectation- more dangerous than contemplated by ordinary consumer who purchaser with common knowledge of the community

- Connecticut court:o Feasible alternative design may be a factor to consider in determining whether a product is defective,

but it is not a requirement o Some products may be unreasonably dangerous even though there is no feasible alternative designo A jury may be able to infer a defect in some circumstances without expert testimony as to alternative

design o Not comfortable with adopting the Restatement 3d

- Causation:o Cause in in fact remains a required element of proofo P must prove that the defect existed when the product left the manufacturer’s/seller’s control o P must prove that the defect was a cause in fact of the injury

62

Page 63: Torts 2 Case Briefs

- Difficult cases for P’s:o Warnings

Rebuttable presumptiono Crashworthiness cases- trying to figure out how much of the injury could have been avoided had the

product been property designed (car airbags, ex. ) o Drug cases- who manufactured?- DES cases- how to decide who manufactured the drugs?

- Legal Causeo P must be foreseeable, but if hindsight test, danger need not be foreseeable (at lease if the use was

foreseeable)o However, even then, court is likely to use foreseeability in some way, as in “now that we know the risk,

is the risk one that would be considered reasonably foreseeable in the future- By standers:

o Elmore v. American Motors Corp. Bystanders injured by product defect may bring strict tort liability claims. Still must pass test of foreseeability Have to prove causal connection

Legal cause- foreseeability Cause in fact- but for test

o In most states, strict liability applies to any seller of a defective product, as long as the seller is in the business of selling the product

Includes manufacturers, wholesalers, and retailerso A few states apply strict liability only to manufacturers (must be within the jurisdiction of the court and

not bankrupt)- Pure economic loss

o General rule: P may not recover damages in tort for physical damage to the product itself. (Ordinarily such a claim sounds in warranty or contract)

o If you buy a computer and it blows up, you can’t sue for the damage to the computer. However, if it burns the house down, you can sue for the damage done to the house. Have to sue under warranty or contracts instead for the damage to the computer.

o P may recover for physical damage to other property caused by o Saratoga Fishingo Note 1- Majority view: cannot recover in tort for damage to the product itsef o Note 2- tort recovery is allowed for physical damage to other property and economic loss caused by

such physical damage or personal injury - Defenses based on consumer conduct:

o Daly v. General Motors Corp. Can contributory negligence of P reduce P’s recovery in a strict liability products claim?

YES!!!!!!o Comparative o

Torts Review- Strict liability

o Animalso Abnormally dangerous activities

(note: structure is the same as strict products liability- what is the standard that pploes: strict liability or negligence? If negligence (i.e., not abnormally dangerous), then normal duty-breach-cif-legal cause- damages approach. If strict liability is the appropriate rule, then that establishes

63

Page 64: Torts 2 Case Briefs

the standard (in place of duty of reasonable care) and breach simply becomes a matter of establishing that the standard of strict liability applies to these facts)

o Products liability: Duty: Determine which theory(ies) apply. Generally rules apply to users and bystanders (unless

determined to be unforeseeable as matter of legal cause) as plaintiffs. As to defendants, generally manufacturers, wholesalers, retailers

Negligence Strict liability

o Manufacturingo Design

402-A Consumer risk expectation Risk/utility in hindsight

Restatement 3d

64